Sie sind auf Seite 1von 158

SAT II Biology E/M Dr Haitham AbdAllah

0100 36 777 19

Photosynthesis
Questions 1-5

Experimental evidence from a study that compares the rate of photosynthesis with the absorption of light by
various pigments is illustrated in the graph below.

1. The data shown in the graph could have been determined by measuring the production of which of the
following?
(A) Ribulose biphosphate
(B) Water
(C) Oxygen
(D) Carbon dioxide
(E) Chlorophyll

2. The greatest rate of photosynthetic activity is driven by absorption of light energy in which of the following
ranges?
(A) 400-450 nm
(B) 450-520 nm
(C) 500-600 nm
(D) 630-660 nm
(E) 660-710 nm

3. According to the data, which of the following colors corresponds to the lowest rate of photosynthesis?
(A) Ultraviolet
(B) Violet
(C) Blue
(D) Yellow
(E) Orange

4. According to the information in the graph, which of the following wavelengths of light would cause a
floating aquatic plant to attract the fewest oxygen-seeking bacteria?
(A) 450 nm
(B) 500 nm
(C) 550 nm

1
Classified Real Questions
SAT II Biology E/M Dr Haitham AbdAllah
0100 36 777 19

(D) 600 nm
(E) 650 nm

5. According to the graph, which of the following colors would cause the greatest rate of CO 2 uptake by a
plant?
(A) Ultraviolet
(B) Violet
(C) Green
(D) Yellow
(E) Orange

Questions 6-8

1937 Robin Hill showed that isolated chloroplasts can carry out a portion of photosynthesis if they are
illuminated and provided with an electron acceptor. The "Hill reaction" is performed using a blue dye (DPIP)
in place of NADP because the dye changes from blue to colorless as it accepts an electron. Five test tubes were
wrapped in aluminum foil and set up as follows.

Tubes 1 to 4 were then exposed to light. The disappearance of the blue color as the reaction proceeded was
measured with a spectrophotometer. The following data were obtained.

6. The purpose of tube 5 was to control for


(A) Changes not due to the light exposure
(B) Changes in the absence of dye
(C) Time
(D) pH
(E) Temperature

7. In a graph of the percent of blue dye remaining versus exposure time, which of the following would indicate
the rate of the Hill reaction at the bright light intensity in tube 2?
(A) The origin

2
Classified Real Questions
SAT II Biology E/M Dr Haitham AbdAllah
0100 36 777 19

(B) The area under the curve


(C) The slope of the curve
(D) The amount of blue color remaining at the end of the reaction
(E) The maximum percentage of blue color remaining minus the minimum percentage of blue dye remaining

8. Isolated chloroplasts are known to break down over time. Data from the experiment that support this
statement include which of the following ?

I. In bright light, more blue dye became colorless in the first 30 seconds of the experiment than in the
last 30 seconds
II. After 30 seconds, more blue color disappeared in bright light than in dim light
III. The reaction cannot take place after 120 seconds
(A) I only
(B) II only
(C) III only
(D) I and III only
(E) I, II, and III

9. How does the rate of reaction relate to the intensity of light?


(A) The two are directly related
(B) The two are inversely related
(C) No relation exists between the two
(D) The two are independent for the first 90 seconds
(E) The two are inversely related early in the experiment and directly related toward the end of the experiment

Answers :
1.C
2.A
3.D
4.C
5.B
6.A
7.C
8.A
9.A

3
Classified Real Questions
SAT II Biology E/M Dr Haitham AbdAllah
0100 36 777 19

Cell division & protein synthesis

Questions 1-3 :

(A) Anaphase II
(B) Metaphase I
(C) Prophase II
(D) Metaphase II
(E) Prophase I

1. Stage of meiosis during which recombination of genetic material occur


2. Stage of meiosis during which pairs of homologous chromosomes align at the
center of the cell
3. Stage of meiosis during which sister chromatids are separated

4.

4
Classified Real Questions
SAT II Biology E/M Dr Haitham AbdAllah
0100 36 777 19

5. Meiosis in diploid organisms results in which of the following?


(A) The production of haploid cells
(B) The generation of new alleles
(C) The formation of a zygote
(D) The transmission of adaptive phenotypes
(E) The creation of triploid gametes

6. Which of the following best describes what is meant by the term ‘’ diploid ‘’
(A) Containing double-stranded DNA
(B) Containing only one copy of each chromosome
(C) Containing pairs of structurally similar choromosomes
(D) Containing pairs of chromosomes with identical DNA sequences
(E) Containing haphazardly arranged genes

7. A cytologist studying onion cells notes that after 12 hrs, the amount of DNA in a
cell has doubled. Which of the following is the best explanation for this
observation ?

(A) Nucleotides have diffused inot the cell against concentration gradient
(B) Metaphase has begun, concentrating chromosomes in the center of the cell
(C) Protein synthesis has stopped, resulting in an overproduction of DNA
(D) New plasma membranes have been formed during cytokinesis
(E) The S phase of the cell has occurred

8. The codon GUG in messenger RNA has been transcribed from which of the
following sequences in DNA ?
(A) GAG
(B) CAC
(C) TAT
(D) CTC
(E) ATA

9. Humans doNOT obtain energy from eating cellulose because ?

(A) Breaking down cellulose into glucose somply requires too much energy
(B) Cellulose is incorporated into the cell walls and hence none is left to burn
(C) The required enzyme necessary to hydrolyze cellulose is lacking
(D) Only alpha glucose may enter the glycolytic pathway
(E) The combustion of glucose is not exergonic

10. The main structural framework for the plasma membrane is provided by which of the following
molecules ?
(A) Water
(B) Nucleotides
(C) Monosaccharides

5
Classified Real Questions
SAT II Biology E/M Dr Haitham AbdAllah
0100 36 777 19

(D) Amino acids


(E) Phospholipids

11. In both photosynthesis and respiration, protons are pumped across the membrane
during

(A) Chemiosmosis
(B) Photolysis
(C) Photorespiration
(D) Carbon fixation
(E) Glycolysis

Questions 12- 15 :

(A) DNA
(B) tRNA
(C) mRNA
(D) rRNA
(E) RNA polymerase

12. Translated to synthesize protein


13. Transports amino acids during protein synthesis
14. Passed on to progeny cells during cell division
15. . Includes a structure called ‘’ anti-codon’’

16. All of the following are needed for photosynthesis except ?

(A) Light
(B) Glucose
(C) Chlorophyll
(D) Water
(E) Carbon dioxide

17. If one ribose molecule is bonded to one adenine molecule and one phosphate molecule,
we would have a

(A) Ribosome
(B) Nucleotide
(C) Nucleic acid
(D) ATP
(E) ADP

18. Which of the following organelles would be present in a eukaryote but NOT in a
prokaryote ?
I. Nucleus
II. Mitochondria
III. Ribosome

(A) I only

6
Classified Real Questions
SAT II Biology E/M Dr Haitham AbdAllah
0100 36 777 19

(B) II only
(C) I and II only
(D) II and III only
(E) I , II , III

19. What type of molecule is depicted in the diagram above ?


(A) Protein
(B) Carbohydrate
(C) Lipid
(D) Nucleic acid
(E) Starch

20. If we were to continue adding to this molecule to increase in size, to which atom would
the next portion be bonded?
(A) 1
(B) 2
(C) 3
(D) 4
(E) 5

21. The synthesis of this molecule also results in the production of

(A) Carbon dioxide


(B) Water
(C) Lipid
(D) ATP
(E) NaOH

Questions 22-25 : refer to the diagram showing how a bacterium makes a protein from the instructions in
DNA

7
Classified Real Questions
SAT II Biology E/M Dr Haitham AbdAllah
0100 36 777 19

22. The ribosome is represented by which labeled structure ?


(A) 1
(B) 3
(C) 5
(D) 6
(E) 7

23. Messenger RNA is represented by which labeled structure ?


(A) 1
(B) 2
(C) 4
(D) 5
(E) 6

24. A hydrogen bond is represented by which labeled structure ?


(A) 1
(B) 2
(C) 3
(D) 4
(E) 5

25. Which of the labeled structure contains both protein and RNA ?
(A) 1
(B) 2
(C) 4
(D) 5
(E) 6

Questions 26-30 :

8
Classified Real Questions
SAT II Biology E/M Dr Haitham AbdAllah
0100 36 777 19

(A) Hydrolysis
(B) Dehydration synthesis
(C) Ionization
(D) Phosphorylation
(E) Fermentation

26. H2CO3 → H+ +HCO3-


27. Glucose → alchol + carbon dioxide
28. Fat + water → fatty acids + glycerol
29. Glucose + fructose → sucrose + water
30. Polypeptide + water → amino acids

Questions 31-33 :

31. Which of the following doesNOT contain membrane of its structure?


(A) Chromosome
(B) Mitochondrion
(C) Golgi body
(D) Chloroplast
(E) Endoplasmic reticulum

32. In animals, mitosis is LEAST likely to play a significant role in which of the following ?

(A) Reproduction of cells


(B) Repair of damaged tissue
(C) Growth of embryonic tissues
(D) Regenration of limbs
(E) Reduction of chromosomes number

33. An organism possessing the following characteristics is observed : asexual reproduction,


eukaryotic, heterotrophic, chitinous cell walls. The organism is most likely belongs to
which of the following taxonomic groups?
(A) Bacteria
(B) Archaea
(C) Fungi
(D) Plantae
(E) Animalia

34.

35. Of the following, the best indicator of the rate of photosynthesis in green plants is the rate at which
(A) Water is produced

9
Classified Real Questions
SAT II Biology E/M Dr Haitham AbdAllah
0100 36 777 19

(B) Carbon dioxide is given off


(C) Oxygen is consumed
(D) Sugar is consumed
(E) Carbon dioxide is consumed

Questions 36-37 :

36. A somatic cell from a certain organism has a diploid number of 24 When the cell reproduces by cell
division how many chromosomes will each daughter cell have

(A) 12
(B) 23
(C) 24
(D) 46
(E) 48

37. Which of the following structures is found in both prokaryotic and eukaryotic cells ?
(A) Mitochondrion
(B) Nucleus
(C) Chloroplasts
(D) Endoplasmic reticulum
(E) Ribosome

Questions 38-41 :
A group of students extracted an enzyme from rutabaga stems and exanuned its rate of reaction at different
temperatures and different substrate concentrations. The results are shown in the tables

10
Classified Real Questions
SAT II Biology E/M Dr Haitham AbdAllah
0100 36 777 19

38. From the data in table II, it can be inferred that the rate of reaction.
(A) Is independent of substrate concentration.
(B) Increases linearly with substrate concentration.
(C) Is inversely proportional to substrate concentration.
(D) Is proportional to substrate concentration only at low substrate concentrations.
(E) Is proportional to substrate concentration only at high substrate concentrations.

39.The data on the effects of temperature would be best represented by which of the following
graphs?

40.The effects of temperatures greater than 40°C on enzyme activity can best be explained by which of the
following?

(A) The substrate becomes more soluble in a warmer solution.


(B) The product of the reaction precipitates in a warmer solution.
(C) The enzyme catalyzes the reaction too rapidly to make the product.
(D) The peptide bonds are broken by the high temperatures.
(E) The high temperatures denature the enzyme, inactivating it.

41.Which of the following is true about the effect of temperatures between 25 °C and 35 °C enzyme activity?

I. The rate of diffusion increases with temperature, allowing substrate and enzyme molecules to encounter one
another more frequently .
II. The reaction catalyzed by the enzyme proceeds more rapidly at higher temperatures.
III. Increased temperature hydrolyzes peptide bonds, causing the formation of an enzyme fragment that works
more efficiently.
(A) I only
(B) II only.
(C) III only.
(D) I and II only.
(E) I and III only

11
Classified Real Questions
SAT II Biology E/M Dr Haitham AbdAllah
0100 36 777 19

42. The plasma membrane of eukaryotic cell function in the


(A) Synthesis of lipids and steroid hormones
(B) Polymerization of proteins
(C) Breakdown of hydrogen peroxide and ammonia
(D) Release of energy from carbohydrates and fats
(E) Regulation of nutrients entering and leaving the cell

43. If a trait is controlled by a single gene pair, then the genotype of an individual carrying
identical alleles for that trait is referred to as
(A) Hybrid
(B) Dominant
(C) Recessive
(D) Haploid
(E) Homozygous

44. Lactic acid in humans is a product formed as a result of which of the following
processes?
(A) Breakdown of nucleic acids
(B) Breakdown of protein in digestion activity of muscle cells when oxygen supply is
insufficient
(C) Activity of muscle cells when oxygen supply is insufficient
(D) Release of stomach acids during development of an ulcer
(E) Reaction of fats with phosphorus compounds

45. As a result of his work with garden peas, Gregor Mendel proposed all of the following
EXCEPT
(A) Crossing-over
(B) Segregation
(C) Paired hereditary factors
(D) Independent assortment
(E) Dominance

46. Convergent evolution results when two or more species


(A) Become dissimilar after reproductive isolation
(B) Continue to resemble each other through time
(C) Come to resemble each other even though they are phylogenetically dissimilar
(D) Exert a strong selective pressure in each other over time
(E) Are subjected to disruptive selection

47. If a typical animal cell with an intracellular fluid containing 0.9 percent solutes is placed
in distilled water , the cell will

(A) Undergo mitosis


(B) Undergo plasmolysis
(C) Remain unchanged in size
(D) Excrete proteins

12
Classified Real Questions
SAT II Biology E/M Dr Haitham AbdAllah
0100 36 777 19

(E) Swell and burst

48. When during cell division do chromosomes move to opposite poles of the cell?
(A) When the centrioles replicate ..
(B) After the nuclear membrane disintegrates.
(C) Immediately following DNA replication
(D) When the DNA condenses.
(E) Immediately after the centromere splits.

49. The homologous chromosomes in the cell pictured above are engaged in a process called
(A) Cytokinesis
(B) Substitution
(C) Replication
(D) Crossing-over
(E) Translation

50. Crossing over occurs during


(A) Anaphase 1
(B) Telophase 1
(C) Prophase 1
(D) Metaphase 1
(E) Interkinesis

51. A genetically important event of crossing over occurs during


(A) Metaphase II
(B) Telophase 1
(C) Prophase 1
(D) Anaphase 1
(E) Telophase II

52. Which cell division process results in 4 genetically different daughter cells that contain one haploid
set of chromosomes?
(A) Interphase
(B) Somatic cell regeneration
(C) Cell division
(D) Mitosis
(E) Meiosis

13
Classified Real Questions
SAT II Biology E/M Dr Haitham AbdAllah
0100 36 777 19

53. Translation, transcription, and replication take place in this phase of the cell cycle
(A) G1
(B) G2
(C) Metaphase
(D) Anaphase
(E) S

Questions 54–55 refer to the following table :

54. What would be the DNA triplet that would code for the addition of arginine to a polypeptide?
(A) C–G–A
(B) G–C–T
(C) C–C–T
(D) T–A–C
(E) G–C–U

55. Which amino acid would be attached to a polypeptide chain if the amino acid was attached to the
anticodon A–A–A?
(A) phenylalanine
(B) lysine
(C) valine
(D) a stop codon
(E) cannot be determined from the information given

56. Which task requires recombinant DNA technology?


(A) Creating bacteria that produce human growth hormone
(B) Creating a polyploidy cherry tree
(C) Crossing two types of orange trees to create new orange fruits
(D) Crossing a horse with a donkey to breed a mule
(E) Paternity testing

57.

A somatic cell of the common octopus (Octopus vulgaris) has 56 chromosomes. What number of
chromosomes will an octopus gamete contain?
(A) 14
(B) 28

14
Classified Real Questions
SAT II Biology E/M Dr Haitham AbdAllah
0100 36 777 19

(C) 56
(D) 112
(E) 168

58.
A genetic mutation results in a DNA sequence that continues to synthesize the same protein, at the same
rate as a nonmutated sequence. This mutation would be BEST classified as a
(A) deletion
(B) insertion
(C) missense mutation
(D) nonsense mutation
(E) silent mutation

59.
Which of the following typifies cytokinesis during mitosis?
(A) Crossing-over
(B) Formation of the spindle
(C) Formation of tetrads
(D) Synapsis
(E) Division of the cytoplasm

Questions 60-63:

60. During ‘’S-phase’’ of the cell cycle, C.elegans cells are


(A) Growing larger
(B) Dividing into two new cells
(C) In a period of dormancy

15
Classified Real Questions
SAT II Biology E/M Dr Haitham AbdAllah
0100 36 777 19

(D) Copying DNA and organelles


(E) Synthesizing large amounts of ATP

61. During which of the following phases does the nuclear membrane of the cell start to
break apart so that chromosomes will be able to move to opposite sides of the dividing
cell?
(A) Interphase
(B) Prophase
(C) Metaphase
(D) Anaphase
(E) Telophase

62. At the very end of mitosis, two daughter cells will be created by a process that pinches
one cell in two. This process of pinching is called
(A) Cytokinesis
(B) Cytolysis
(C) Cytosolic pinching
(D) Endocytosis
(E) Exocytosis

63. During all the following phases of the cell cycle, C.elegans will have its DNA in a
replicated state, EXCEPT
(A) G1
(B) S
(C) G2
(D) M
(E) All of interphase

64. A spermatocyte produces the following four sperm cells

These cells are the result of nondisjunction during which of the following phases?
(A) Interphase (Gl or G2)
(B) Interphase (S)
(C) Mitosis
(D) Meiosis I
(E) Meiosis II

65. The nitrogenous base, adenine, is found in which three of the following
(A) Proteins, chlorophyll, and vitamin A
(B) Protein, ATP, and DNA
(C) ATP, DNA, and RNA
(D) Chlorophyll, ATP, and DNA
(E) Proteins, carbohydrates, and ATP
66.

16
Classified Real Questions
SAT II Biology E/M Dr Haitham AbdAllah
0100 36 777 19

A protein synthesized in the cytoplasm and transported to the plasma membrane. Which of
the following summarizes the protein’s pathway in the cell?
(A) Smooth endoplasmic reticulum → nucleus → vesicle → plasma membrane
(B) Plastid → rough endoplasmic reticulum → plasma membrane
(C) Nucleus → vesicle → rough endoplasmic reticulum → plasma membrane
(D) Smooth endoplasmic reticulum → microfilament → vesicle → plasma membrane
(E) Rough endoplasmic reticulum → golgi complex → vesicle → plasma membrane
Questions 67-68

Haemophilus influenzae DNA is made up of about 1.800 kilobase pairs organized into about 1,750 genes. The
DNA is rich in cytosine.

67. If the genes of H. influenzae were all located on a single strand of the double helix and were adjoining one
another end to end, an average gene would include approximately how many bases?
(A) 10
(B) 100
(C) 1,000
(D) 10,000
(E) 1 ,000,000

68. The DNA of H. influenzae is enriched in two nitrogenous bases. One of these is.
(A) Adenine.
(B) ATP.
(C) Guanine.
(D) Thymine.
(E) Uracil

17
Classified Real Questions
SAT II Biology E/M Dr Haitham AbdAllah
0100 36 777 19

Questions 69-72: refer to the following illustration and information.

The cell cycle is a series of events in the life of a dividing eukaryotic cell. It consists of four stages: G 1, S,
G2, and M. The duration of the cell cycle varies from one species to another, and from one cell type to another.
The G1 phase varies the most. For example, embryonic cells can pass through the G 1 phase so quickly that it
hardly exists, whereas neurons are arrested in the cell cycle and do not divide.

69.During which phase do chromosomes replicate?


(A) G1
(B) S
(C) G2
(D) M
(E) Cytokinesis
70.. In mammalian cells, the first sign of prophase is the
(A) appearance of chromosomes
(B) separation of chromatids
(C) disappearance of the nuclear membrane
(D) replication of chromosomes
(E) crossing over of homologous chromosomes
71.. Mitosis occurs in all of the following types of cells EXCEPT
(A) epidermal cells
(B) hair cells
(C) red blood cells
(D) pancreatic cells
(E) kidney cells
72.Since neurons are destined never to divide again, what conclusion can be made?

18
Classified Real Questions
SAT II Biology E/M Dr Haitham AbdAllah
0100 36 777 19

(A) These cells will go through cell division.


(B) These cells will be permanently arrested in the G1 phase.
(C) These cells will be permanently arrested in the G2 phase.
(D) These cells will quickly enter the S-phase.
(E) The duration of the cell cycle will be long

19
Classified Real Questions
SAT II Biology E/M Dr Haitham AbdAllah
0100 36 777 19

Answer Key : Cell Division & protein synthesis

1. E
2. B
3. A
4. A
5. A
6. C
7. E
8. B
9. C
10. E

11. A
12. C
13. B
14. A
15. B
16. B
17. B
18. C
19. A
20. D

21. B
22. C
23. C
24. B
25. D
26. C
27. E
28. A
29. B
30. A

31. A
32. E
33. C
34. B
35. E
36. C
37. E
38. D
39. A
40. E

41. B
42. E
43. E
44. C

20
Classified Real Questions
SAT II Biology E/M Dr Haitham AbdAllah
0100 36 777 19

45. A
46. C
47. E
48. E
49. D
50. C
51. C
52. E
53. E
54. B
55. A
56. A
57. B
58. E
59. E
60. D
61. B
62. A
63. A
64. D
65. C
66. E
67. C
68. C
69. B
70. A
71. C
72. B

21
Classified Real Questions
SAT II Biology E/M Dr Haitham AbdAllah
0100 36 777 19

Protein synthesis

Questions 1-3 : refer to the diagram below, which represents several stages of protein synthesis.
The structures are not drawn to scale.

1. Which of the following is represented by structure 1?


(A) A chloroplast.
(B) A ribosome.
(C) The smooth endoplasmic reticulum.
(D) A chromosome.
(E) The nucleolus.

2. Which of the following is represented by structure 2?


(A) A three-carbon sugar.
(B) A peptide containing three amino acids.
(C) A base triplet of DNA.

22
Classified Real Questions
SAT II Biology E/M Dr Haitham AbdAllah
0100 36 777 19

(D) A base triplet of transfer RNA.


(E) A base triplet of messenger RNA.

3. Which of the structures in the diagram above represents a polypeptide?


(A) 1
(B) 2
(C) 3
(D) 4
(E) 5

Answers :
1. B
2. E
3. E

23
Classified Real Questions
SAT II Biology E/M Dr Haitham AbdAllah
0100 36 777 19

Bio-Technology
1.Restriction enzymes can be used as.tools to move specific segments of DNA from one plasmid to another
Which of the following enables the specific DNA segment to be inserted into. The new plasmid

(A) Complementary ends produced by the enzymes


(B) Replacement of thymine with uracil in RNA
(C) Adding ATP to the restriction digest incubation
(D) Poly A-tails of newly replicated proteins
(E) Adding reverse transcriptase

2. Once a plasmid has incorporated specific genes, such as the gene coding for the antibiotic ampicillin, into its
genome, the plasmid may be cloned by
(A) inserting it into a virus to generate multiple copies
(B) treating it with a restriction enzyme in order to cut the molecule into small pieces
(C) inserting it into a suitable bacterium in order to produce multiple copies
(D) running it on a gel electrophoresis in order to determine the size of the gene of interest
(E) infecting it with a mutant cell in order to incorporate the gene of interest

Answers:
1.A
2.C

24
Classified Real Questions
SAT II Biology E/M Dr Haitham AbdAllah
0100 36 777 19

Genetics

1. A particular characteristic in human is inherited as an autosomal


recessive trait . if a man with the trait and a woman without it have three
sons without the trait, which of the following can be concluded ?
(A) The children are heterozygous for the trait
(B) The woman is homozygous recessive for the trait
(C) The man is heterozygous for the trait
(D) The fourth child will have the trait
(E) The trait is sex-linked

2. Color blindness is inherited as a sex-linked recessive trait . A woman


with normal color vision whose father was color-blind marries a man
with normal color vision. What is the probability that their first child will
be a son who is color-blind ?
(A) 1/10
(B) ¼
(C) ½
(D) 9/16
(E) ¾

3.In a certain plant species, the trait tall T is dominant to dwarf t and the color red R
is dominant to white r .To determine whether a particular tall red plant is
heterozygous or homozygous for both characteristics , the plant would be best
crossed with which of the following?
(A) TTRR
(B) TtRr
(C) ttRR
(D) TTrr
(E) ttrr

4.At what stage of mitosis in an animal cell do chromosomes move towards the pole
of spindle?
(A) As the DNA in the chromosomes replicates
(B) When the centrioles first appear
(C) As soon as the nuclear membrane breaks down
(D) Right after the centromeres in each chromatid pair separate
(E) Right after cytokinesis occur

Questions 5-9 :
(A) 2
(B) 4
(C) 16
(D) 25
(E) 50

25
Classified Real Questions
SAT II Biology E/M Dr Haitham AbdAllah
0100 36 777 19

5.The number of different phenotypes possible for the progeny of the cross Aa X aa
6.The percent probability that expectant parents who already have two daughters will
have another daughter
7.The smallest number of generations required to obtain 9:3:3:1 ratio of phenotypes
starting with parents having the progeny AABB and aabb
8.The number of different gametes that could be produced by an individual with the
genotype AaBb
9.The number of chromatids in a cell at prophase 1 of meiosis for Drosophila
melanogaster, whose haploid chromosome number is 4

10. Two mature plants that are each 12 centimeters in height are crossed and produce
offspring ranging from 4 to 18 centimeters in height when mature. Of the following,
which is most likely explanation for these results?

(A) Tallness is dominant over shortness


(B) Multiple genes affect heights in plants
(C) Height is a sex-linked trait in plants
(D) Non-disjunction has occurred
(E) A mutation has occurred

11.Which of the following are the genotypes of a couple that have four children, each
with a different blood type?

(A) AA and BO
(B) AB and BO
(C) AO and AB
(D) AO and BO
(E) AB and OO

Questions 12-14 :

Tay Sachs disease is a genetic defect controlled by a single pair of alleles. The recessive allele, when
homozygous causes the disease , which is characterized by the inability to synthesize a particular
enzyme. For the three families depicted below, circles represent females and squares represent males.
Symbols for individuals who have Tay-Sachs disease are shaded. Testing has shown that person I and
person III are carriers of the Tay-sachs allele and that person II is not .

26
Classified Real Questions
SAT II Biology E/M Dr Haitham AbdAllah
0100 36 777 19

1.

2.

3.

Questions 15-17 :

refer to an- experiment in which liver cells from a live culture are mechanically ruptured to analyse the
cell componentf. The resulting suspension was centrifuged several times in succession to produce layers
in which certain cell organelles predominated, as shown in the diagram below

27
Classified Real Questions
SAT II Biology E/M Dr Haitham AbdAllah
0100 36 777 19

4. Upon analysis, Layer A was found to have the highest proportion of


DNA, indicating that the layer contained the major portion of the
(A) Mitochondria
(B) Nuclei
(C) Ribosomes
(D) Lysosomes
(E) Endoplasmic reticulum

5. Upon analysis, Layer D was found to have the highest rate of oxygen
uptake. This layermost likely contained
(A) Mitochondria
(B) Nuclei
(C) Ribosomes
(D) Lysosomes
(E) Endoplasmic reticulum

6. Upon analysis, layer C had the greatest concentration of RNA and was
probably made up mostly of
(A) Mitochondria
(B) Nuclei
(C) Ribosomes
(D) Lysosomes
(E) Plasma membrane

Questions 18-22 :
(A) Meiosis

28
Classified Real Questions
SAT II Biology E/M Dr Haitham AbdAllah
0100 36 777 19

(B) Mitosis
(C) Fertilization
(D) Mutation
(E) Crossing-over

7. Process by which the number of chromosomes is reduced


8. Process by which the number of chromosomes is restored
9. Process by which the number of chromosomes is kept constant in all the
cells of an organism
10. Process by which chromosomes exchange materials
11. Process by which genes change in composition

Questions 23-25 :

12.

13.

14.

Questions 26-28 :

29
Classified Real Questions
SAT II Biology E/M Dr Haitham AbdAllah
0100 36 777 19

15.

16.

17.

Questions 29-32 :

30
Classified Real Questions
SAT II Biology E/M Dr Haitham AbdAllah
0100 36 777 19

18.

19.

20.

21.

31
Classified Real Questions
SAT II Biology E/M Dr Haitham AbdAllah
0100 36 777 19

22.

23.

32
Classified Real Questions
SAT II Biology E/M Dr Haitham AbdAllah
0100 36 777 19

24. Measurements were made of the number of mammalian cells of a certain


cell line growing under in vitro conditions in a liquid medium. The data
are shown in the figure below. What is the length of one cell cycle for
these cells?

(A) 10 hours
(B) 20 hours
(C) 30 hours
(D) 40 hours
(E) 50 hours

36. Chromosomal mutations include all of the following EXCEPT


(A) Deletion
(B) Duplication
(C) Inversion
(D) Translocation
(E) Synapsis

37. A rancher bought a prize bull guaranteed not to have a particular unwanted recessive allele of an
autosomal gene. He crossed the bull to cows who were homozygous for the unwanted allele, with
the result that 3 of the progeny were normal and 2 had the undesired trait. Which of the following
is the best conclusion to be drawn from this result?

(A) The bull is homozygous dominant for the unwanted allele.


(B) The bull is homozygous recessive for the unwanted allele.
(C) The cows are really heterozygous for the unwanted allele.
(D) It is not possible for selective breeding to determine the genotype of an animal, so the original
guarantee was unreasonable
(E) The rancher was cheated; the bull is heterozygous for the unwanted allele

Questions 38-41 :

Refer to the pedigree shown below that illustrates the transmission of trait called albinism, the lack of
pigmentation. For the family tree shown in this pedigree, the trait has only two alleles. A is dominant and
a is recessive

33
Classified Real Questions
SAT II Biology E/M Dr Haitham AbdAllah
0100 36 777 19

38. According to the pedigree, one can correctly conclude that


(A) Most of the family members are albino.
(B) Parents who are normally pigmented can have a child who is albino.
(C) Parents who are both albino can have a normally pigmented child.
(D) Albinism is virally transmitted.
(E) Albino individuals can have normally pigmented brothers but not normally pigmented sisters

39. The genotype of a person who exhibits albinism is correctly expressed by


(A) AAonly.
(B) Aa only.
(C) aa only.
(D) Aa or aa.
(E) AA or Aa.

40. The genotypes of individuals 1 and 2, respectively, are


(A) AA and Aa.
(B) Aa and Aa.
(C) Aa and aa.
(D) AA and aa.
(E) Not determinable from the data

41. If individuals 8 and 9 have another child, the chance that this child will exhibit albinism is.
(A) 0%
(B) 25%
(C) 50%
(D) 75%
(E) 100%

42. In a man showing the recessive phenotype for a particular autosomal trait, what is the expected
percentage of his sperm cells that will carry the recessive allele associated with this trait?
(A) 0%
(B) 25%
(C) 50%
(D) 75%
(E) 100%

43. Alleles A1 A2 , and A3 are each expressed whenever present in a genotype. In the cross A 2 A3 x A1 A2
, what proportion of offspring should express A2?

34
Classified Real Questions
SAT II Biology E/M Dr Haitham AbdAllah
0100 36 777 19

(A) 0%
(B) 25%
(C) 50%
(D) 75%
(E) 100%

44. A testcross is used to determine the genotype of a plant showing the dominant tall trait. Which of the
following probably shows that the plant is heterozygous?
I. All the offspring show the dominant trait.
II. All of the offspring show the recessive trait
III. Some of the offspring are tall.
IV. None of the offspring are short.
(A) I only.
(B) II only.
(C) III only.
(D) IV only.
(E) III and IV

45. If a trait is controlled by a single gene pair, then the genotype of an individual carrying identical
alleles for that trait is referred to as.
(A) Hybrid.
(B) Dominant.
(C) Recessive.
(D) Haploid.
(E) Homozygous.

1. A and a are alleles of one gene, and B and b are alleles of another gene. The two genes are not linked.
If a parent has the genotype AaBb , which gamete types could be produced?

(A)AB only
(B) Aa and Bb only
(C) AB and ab only
(D) Ab and aB only
(E) AB, Ab, aB, ab

2. In sheep, white wool is due to single dominant allele and black wool is due to recessive allele. Two
sheep with white wool are mated and produce a lamb with black wool. If they produce another lamb,
what is the probability that it will have white wool?
(A) 0%
(B) 25%
(C) 50%
(D) 75%
(E) 100%

3. In the cross Ff X Ff in which F is the allele for the dominant trait, the genotype of offspring with
dominant phenotype is expected to be
(A) 0
(B) ¼

35
Classified Real Questions
SAT II Biology E/M Dr Haitham AbdAllah
0100 36 777 19

(C) ½
(D) ¾
(E) 1

Questions 49-52 :

Self-pollinating plant with orange flowers and alternating leaf arrangement produces. 47 plants with red
flowers alternating leaves, 103 plants with orange flowers and alternating leaves, and S 1 plants with
yellow flowers alternating leaves

4. The allele for for yellow flower color is


(A) Recessive
(B) Dominant
(C) Incompletely dominant
(D) Incompletely recessive
(E) Co-dominant
5. If a yellow-flowered offspring were self-pollinated, what would tho resulting plants' flowers look
like?
(A) 100% yellow
(B) 100% red
(C) 100% orange
(D) 50% yellow , 50% red
(E) 70% red 25% yellow

6. The genotype of the original parent plant is


(A) Heterozygous for both flower color and leaf arrangement
(B) Homozygous for both flower color and leaf arrangement
(C) Homozygous for flower color and heterozygous for leaf arrangement
(D) Heterozygous for flower color and homozygous for leaf arrangement
(E) Heterozygous for flower color but unable to detenninc genotype for leaf arrangement

7. Suppose the original parent plant is cross pollinated with a plant that has red flowers and
and non-alternating leaves. All of the resulting offspring have non-alternating leaves. Which of the
following is/ are true?
I. None of the offspring have orange flowers
II. The allele for alternating leaves is recessive
III. Approximately half of the offspring have red flowers
(A) I only
(B) II only
(C) III only
(D) I and III only
(E) II and III only

Questions 53- 55 :
refer to the following individuals.
(A) A hybrid between two species.
(B) A genetically engineered individual.
(C) A purebred individual.

36
Classified Real Questions
SAT II Biology E/M Dr Haitham AbdAllah
0100 36 777 19

(D) An asexually reproducing individual.


(E) An individual showing a new mutation

53. A male donkey with 62 chromosomes is bred to a female horse with 64 chromosomes, producing
an offspring called a mule with 63 chromosomes
54. A rooster of the white leghorn breed fertilizes a hen of the same breed. Each chick produced is also
a white leghorn
55. A yeast cell multiplies through budding

56. Turner's Syndrome (XO) is a chromosomal disorder that can be diagnosed by


(A) Pedigree analysis
(B) Biochemical analysis
(C) Punnett square analysis
(D) Karyotyping
(E) Blood type analysis

Questions 57-59 :
In cross the gene for leaf color has two alleles. The allele for purple leaf
color (B) is dominant to that for green leaf color (b). the following data
were gathered for three separate crosses

Parents
Cross Female Male offspring
I Purple X Purple 45 Purple
14 Green

II Purple X Green 60 Purple


0 Green

III Purple X Green 31 Purple


30 Green

57. Based on the data above, the genotype of the parents in cross I are correctly represented by which of
the following?
(A) Bb X bb
(B) Bb X Bb
(C) BB X bb
(D) BB X Bb
(E) BB X BB

58. A cross between any two of the offspring of cross II would yield offspring with a leaf color ratio
similar to that seen in cross
(A) I only
(B) II only
(C) III only
(D) I or II only
(E) II or III only

37
Classified Real Questions
SAT II Biology E/M Dr Haitham AbdAllah
0100 36 777 19

59. The offspring with purple leaves in cross III are expected to be which of the following genotypes ?
(A) 100% BB
(B) 100% Bb
(C) 67% BB and 33% Bb
(D) 50% BB and 50% Bb
(E) 33% BB and 67% Bb

Questions 60-61 :

60.

61.

Questions 62-64 :

(A) Linked genes


(B) Sex-influenced traits
(C) Autosomal recessive traits
(D) Sex-linked traits
(E) Lethal genes

62. The inheritance of blue eye color in humans


63. The occurance of type O blood in children born to parents who each has type A blood
64. The more frequent occurance of pattern baldness in men than in women
65. An individual that has only one X chromosome is genotypically XO. This person
(A) cannot survive

38
Classified Real Questions
SAT II Biology E/M Dr Haitham AbdAllah
0100 36 777 19

(B) will have immature, ambiguous (both male and female) reproductive systems
(C) will be phenotypically female but sterile
(D) does not produce steroid hormones
(E) none of the above

66. Huntington’s disease is a degenerative disorder of the nervous system


that follows an inheritance pattern of autosomal dominance. What is the
probability that a child will inherit Huntington’s disease if one of the
parents has an allele for the disease?
A) 0%
B) 25%
C) 50%
D) 75%
E) 100%

67.
Pattern baldness is a sex-linked recessive trait characterized by hair loss near the hairline and at the
crown of the head. If a woman whose father is bald and a man who is bald have a son, what is the
probability that the son will inherit the allele for pattern baldness? Assume that the woman’s mother
does not carry the allele for pattern baldness.
A) 0%
B) 25%
C) 50%
D) 75%
E) 100%

Questions 68-71 refer to the probabilities below.


Assume that the alleles referred to all assort independently.
(A) 0
(B) l/16
(C) l/4
(D) l/2
(E) 3/4

68. Probability that the genotype Aa will be produced by the parents Aa x Aa


69. Probability that the genotype ccdd will be produced by the parents CcDd x CcDd
70. Probability that the genotype Rr will be produced by the parents Rr x rr
71. Probability that the genotype TTSs will be produced by the parents TTSs x TtSS

72.What is the effect of crossing-over?


(A) Increased genetic variation
(B) Increased number of sex cells
(C) Increased number of body cells
(D) Increased number of chromosomes
(E) Increased rate of cell division

73. In garden peas, a single gene controls stem length. The recessive allele (t) produces short stems when

39
Classified Real Questions
SAT II Biology E/M Dr Haitham AbdAllah
0100 36 777 19

homozygous. The dominant allele (T) produces long stems. A short-stemmed plant is crossed with
a heterozygous long-stemmed plant. Which of the following represents the expected phenotypes of the
offspring and the ratio in which they will occur?

(A) 3 long-stemmed plants : 1 short-stemmed plant


(B) 1 long-stemmed plant : 1 short-stemmed plant
(C) 1 long-stemmed plant : 3 short-stemmed plant
(D) Long-stemmed plants only
(E) Short-stemmed plants only

74. All of the following statements are correct regarding alleles EXCEPT:
(A) Alleles are alternative forms of the same gene
(B) Alleles are found on corresponding loci of homologous chromosomes.
(C) A gene can have more than two alleles.
(D) One allele can be dominant, and the other recessive.
(E) An individual with two identical alleles is said to be heterozygous with respect to that gene.

75. In a species of beetle, one allele of a gene for a certain trait shows complete dominance over the other
allele of the same gene. If two heterozygous beetles are mated, how many of their 160 offspring are expected
to show the recessive trait?

(A) 160
(B) 120
(C) 80
(D) 40
(E) 0
Questions 76-79 refer to the chromosomes depicted below.

40
Classified Real Questions
SAT II Biology E/M Dr Haitham AbdAllah
0100 36 777 19

76.How many traits are coded for in the above chromosomes?


(A) 1
(B) 2
(C) 3
(D) 6
(E) 7

77. The two gametes that contain these chromosomes unite during fertilization and become part of the same
zygote. What would be the individual's phenotype?
(A) Red flower, tall, smooth seeds
(B) Red flower, short, smooth seeds
(C) White flower, short, wrinkled seeds
(D) Red flower, tall, wrinkled seeds
(E) White flower, tall, wrinkled seeds

78.If the offspring resulting from the union of these original gametes were to be self-crossed, what is the
likelihood of each trait appearing in this second generation?

(A) 100% white, 100% tall, 75% wrinkled, 25% smooth


(B) 100% white, l 00% tall, 50% wrinkled, 50% smooth
(C) 100% white, 100% tall, 25% wrinkled, 75% smooth
(D) 50% red, 50% white, 50% tall, 50% short, 75% wrinkled, 25% smooth
(E) 50% red, 50% white, 50% tall, 50% short, 75% wrinkled, 25% smooth

79.What is the process by which these gametes were formed?


(A) Mitosis
(B) Synapsis
(C) Meiosis
(D) Fertilization
(E) Genesis

41
Classified Real Questions
SAT II Biology E/M Dr Haitham AbdAllah
0100 36 777 19

Answer Key : Genetics :

1. A
2. B
3. E
4. E
5. A
6. E
7. A
8. B
9. C
10. B
11. D
12. A
13. A
14. A
15. B
16. A
17. C
18. A
19. C
20. B
21. E
22. D
23. E
24. D
25. D
26. A
27. B
28. D
29. D
30. B
31. B
32. A
33. C
34. D
35. D
36. E
37. E
38. B
39. C
40. B
41. B
42. E
43. D
44. C
45. E

42
Classified Real Questions
SAT II Biology E/M Dr Haitham AbdAllah
0100 36 777 19

46. E
47. D
48. D
49. C
50. A
51. D
52. E
53. A
54. C
55. D
56. D
57. B
58. A
59. B
60. B
61. C
62. C
63. C
64. B
65. C
66. C
67. C
68. D
69. B
70. D
71. C
72. A
73. B
74. E
75. D
76. C
77. E
78. A
79. C

43
Classified Real Questions
SAT II Biology E/M Dr Haitham AbdAllah
0100 36 777 19

Evolution

1. The process by which a new allele of a gene arises within a population is


(A) Fertilization
(B) Independent assortment
(C) Mutation
(D) Genetic drift
(E) Natural selection

2. Comparing which of the following most accurately determines how closely related two species of plants are
?
(A) Fruit color
(B) Branching pattern
(C) Location of flower
(D) DNA nucleotide sequence
(E) Ribosome structure

3. Which of the following are homologous structures?

(A) The wing of a bat and the wing of a butterfly.


(B) The tail of a monkey and the trunk of an elephant.
(C) The foreleg of a dog and the flipper of a whale.
(D) The claws of a cat and the claws of a crab.
(E) The pads on the toes of a tree frog and the suckers on the arms of an octopus

4. Biologists conclude that the close similarity between the DNA sequences of humans and chimpanzees is
evidence of which of the following?
(A) Gene flow between the two species.
(B) Adaptation to similar niches by the two species.
(C) A recent common ancestor of the two species.
(D) Descent of humans from chimpanzees.
(E) Convergent evolution

5. In 1953 Stanley Miller developed a system to study the synthesis of organic compounds under anaerobic
conditions believed to have been present in the primitive atmosphere. Thus, the reaction vessel in his
experiment did NOT contain

(A) O2
(B) H2O
(C) H2
(D) CH4
(E) NH3

6. According to the Hardy-Weinberg principle, which of the following would help to maintain gene frequencies
in a large population over time?
(A) Natural selection.
(B) Mutation.

44
Classified Real Questions
SAT II Biology E/M Dr Haitham AbdAllah
0100 36 777 19

(C) Genetic drift.


(D) Founder effect.
(E) Random mating

7. All of the following provide evidence of the evolutionary relationship between two organisms EXCEPT.
(A) The fossil record.
(B) Radioactive carbon dating.
(C) Comparative anatomy.
(D) Nucleotide sequences.
(E) Amino acid sequences

8. For a mutation to be significant for the process of evolution, it must be present


(A) In diploid cells.
(B) In the nerve cells of an individual.
(C) In gametes.
(D) In all members of the population.
(E) During mitosis in heart cells

9. Which of the following is a characteristic of an individual, rather than a population?


(A) Sex ratio (number of females to number of males)
(B) Age structure. ·
(C) Number of individuals.
(D) Distribution.
(E) Phenotype

10. Prokaryotes are thought to be similar to the flrst forms of cellular life on Earth because; like the flrst forms,
prokaryotes
(A) Lack DNA.
(B) Are parasitic.
(C) Lack a nuclear envelope.
(D) Possess mitochondria.
(E) Are capable of aerobic respiration

11. The ability of an organism to survive in its environment and to produce viable offspring is best described
as which of the following?
(A) Diversity.
(B) Niche.
(C) Phenotype.
(D) Fitness.
(E) Homeostasis

12. A species of Acacia tree in Central America is protected from insects and climbing vines by a species of
ant, which consumes a nectar like substance produced by the tree in specialized chambers. Such a relationship
between two species is a result of :

(A) Convergence
(B) Adaptive radiation
(C) Succession
(D) Co-evolution
(E) Differentiation

Questions 13-15 :

45
Classified Real Questions
SAT II Biology E/M Dr Haitham AbdAllah
0100 36 777 19

Bones of the forelimbs of six vertebrates are shown below. The structure of each forelimb demonstrates
adaptations to the animal's way of life

13. Based on the structure of the


forelimb, which of the following
vertebrates is most likely to use the
forelimbs for grasping?
(A) 1
(B) 2
(C) 4
(D) 5
(E) 6

14. Based on the structure of the


forelimb, which of the following
pairs of vertebrates is most likely to
use the fore limbs in flying?

(A) 1 and 2
(B) 1 and 3
(C) 2 and 3
(D) 2 and 5
(E) 4 and 5

15. Based on the structure of the forelimb, which of the following vertebrates is best adapted for swimming ?

(A) 1
(B) 2
(C) 3
(D) 4
(E) 6

16. Dolphins and sharks have streamlined body structures, making both animals well adapted for life in a
marine environment. This common trait is best explained by which of the following?
(A) Mimicry.
(B) Genetic drift.
(C) Convergent evolution.
(D) Stabilizing selection.
(E) Founder effect

17. Which of the following is a key point in Charles Darwin's theory of evolution?

(A) Biological structures that have become better suited to the environment by their constant use are
most likely to be inherited
(B) Mutations that occur are those that will help future generations fit into their environments
(C) Slight variations among individuals significantly affect the chance.that a given individual will
survive in its environment and be able to reproduce
(D) Genes change in order to help organisms cope with problems encountered within their
environments
(E) Extinction is nature's way to weed out undeserving organisms

46
Classified Real Questions
SAT II Biology E/M Dr Haitham AbdAllah
0100 36 777 19

18. In a small population of animals, only six individuals possess a certain allele. A large storm randomly
skills many individuals of this population, including the six individuals. The gene pool of this population has
been affected by

(A) Natural selection


(B) Speciation
(C) Variation
(D) Genetic drift
(E) Genetic recombination

19. In a population of frogs at equilibrium, the frequency of the dominant allele for green color is 0.8. The
Hardy-Weinberg equation would predict what percentage of this population to have the homozygous
recessive genotype?
(A) 0%
(B) 4%
(C) 32%
(D) 64%
(E) 100%

20.

In a region of undisturbed sedimentary rock, fossils I-5 were found in the layers shown above. Which of the
following is the best interpretation of this fossil record?
(A) Fossil I is younger than fossil 2.
(B) Fossil 2 is 5.000 years old.
(C) Fossil 3 and 4 evolved from fossil 2
(D) Fossil 5 is likely to be a trilobite.
(E) Fossils contains more carbon I4 than fossil I does.

21. In a population of seed-eating birds, the supply of medium-sized seeds decreases and the bird
population become two separate populations: one with larger beaks and one with smaller beaks. This
demonstrates which of the following'?
(A) Sexual selection.
(B) Stabilizing selection.
(C) Directional selection.
(D) Disruptive selection.
(E) Genetic drift.

22. Convergent evolution can result in all of the following EXCEPT


(A) Structures that have similar functions
(B) Behaviors that are similar
(C) Different species that resemble one another

47
Classified Real Questions
SAT II Biology E/M Dr Haitham AbdAllah
0100 36 777 19

(D) Production of a single species from two originally different species


(E) Niches that arc similar

23. Of the following, the determining factor in natural selection is the number of individuals that
(A) Survive.
(B) Are produced.
(C) Migrate.
(D) Reproduce.
(E) Mutate.

24. Which of the following statements does NOT describe a part of the process of natural selection in animals?
(A) Most organisms produce more offspring than can actually survive to reproductive age.
(B) The individuals that are best adapted to the environment survive and reproduce.
(C) There is a struggle for existence.
(D) The individuals in a given species vary.
(E) Acquired traits can be passed on to offspring

25. The model of.evolutionary change in which long periods of species stability are interrupted by brief rapid
change is
(A) Genetic isolation
(B) Hardy-Weinberg equilibrium
(C) Acquired characteristics theory
(D) Punctuated equilibrium
(E) Gradualism

26. In the speculation concerning the origins of life, one theory states that purines, pyrimidines, sugars, and
phosphate combined to form
(A) Nucleotides
(B) Nucleosides
(C) Carbohydrates
(D) Fats
(E) Proteins

27. All of the following provide evidence for the process of evolution EXCEPT
(A) Vestigial organs
(B) Homologous structure
(C) Embryological development
(D) Nucleotide sequences
(E) Identical twins

28. Which of the following correctly explains how a favorable genetic trait can increase in frequency in a
population?
(A) Lamarck’s principle
(B) Natural selection
(C) Adaptive radiation
(D) Genetic recombination
(E) Segregation of alleles

48
Classified Real Questions
SAT II Biology E/M Dr Haitham AbdAllah
0100 36 777 19

29. Natural selection refers to all of the following EXCEPT


(A) Individual organisms differ from one another.
(B) Competition exists between individuals.
(C) The best-adapted organisms arc most likely to survive.
(D) The best-adapted organisms are most likely to reproduce.
(E) The traits an organism acquires in its lifetime are passed down to its off spring

30. Two distinctly different beak sizes occur in a single population of finch called the black-bellied
seedcracker. These birds live in an isolated region in West Africa. The oldest inhabitants of the region
remember that all these finches used to have the same length beak. This change in the population is shown
by this graph. The best explanation for the change in beak length is

(A) Mutation
(B) Stabilizing selection
(C) convergent evolution
(D) Genetic drift
(E) Disruptive selection

31. The theory that there are short periods of rapid evolution followed by long periods with little or no
evolution is called
(A) Gradualism
(B) Lamarckism
(C) Punctuated equilibrium
(D) Natural selection
(E) Random speciation

32. Of the following, evolutionary fitness is best characterized by


(A) Changes in the gene pool of a population
(B) An organism’s ability to contribute to the next generation’s gene pool
(C) The formation of a new species
(D) Increasing complexities in the features of a species
(E) The development of more humanlike traits

33. The introduction of a new organism into a population can create


(A) New mutations

49
Classified Real Questions
SAT II Biology E/M Dr Haitham AbdAllah
0100 36 777 19

(B) New selection pressure


(C) Adaptive radiation
(D) Genetic drift
(E) Biological magnification

34. Which of these form the basis of natural selection?


I. Random mating
II. Limited environmental resources
III. Stable genotype frequencies over time
A. I only
B. II only
C. III only
D. I and II
E. II and III

35.
A recessive allele accounts for 60% of the alleles of a gene in a population. What proportion of the individuals
in this population are
heterozygous?
A. 0.16
B. 0.24
C. 0.36
D. 0.48
E. 0.60

36.
Which of the following best explains why a population is described as the evolutionary unit?
(A) Genetic changes can only occur at the population level.
(B) The gene pool in a population remains fixed over time.
(C) Natural selection affects individuals, not populations.
(D) Individuals cannot evolve, but populations can.
(E) Most changes in a population's gene pool do not result in evolution.

37.
Which of these individuals is most likely to be successful in an evolutionary sense?
A) a reproductively sterile individual who never falls ill
B) an organism that dies after 5 days of life but leaves 10 offspring, all of whom survive
to reproduce
C) a male who mates with 20 females and fathers 1 offspring
D) an organism that lives 100 years and leaves 2 offspring, both of whom survive to
reproduce
E) a female who mates with 20 males and produces 1 offspring

38.
One of the following people set up a closed system to mimic Earthʹs early atmosphere and
discharged electrical sparks through it. A variety of organic compounds common in
organisms were formed. Who did this?
(A) Stanley Miller
(B) Jakob Berzelius

50
Classified Real Questions
SAT II Biology E/M Dr Haitham AbdAllah
0100 36 777 19

(C) Friedrich Wohler


(D) Hermann Kolbe
(E) August Kekulé

39.
Which of these evolutionary agents is most consistent at causing populations to become better suited to their
environments over the course of generations?
A) Mutation
B) Non-random mating
C) Gene flow
D) Natural selection
E) Genetic drift

40.
Each of the following has a better chance of influencing gene frequencies in small populations than in large
populations, but which one most consistently requires a small population as a precondition for its occurrence?
A) Mutation
B) Non-random mating
C) Genetic drift
D) Natural selection
E) Gene flow

Questions 41-43 :

refer to the following experiment. A group of 100 female Daphnia, small crustaceans known as water fleas,
were placed in one of three culture jars of different sizes to determine their reproductive rate. The graph below
shows the average number of offspring produced per female each day in each jar of pond water

51
Classified Real Questions
SAT II Biology E/M Dr Haitham AbdAllah
0100 36 777 19

41.What is the total number of offspring produced in the 0.5-liter jar on the twentieth day, assuming all
survive?
(A) 2
(B) 4
(C) 50
(D) 200
(E) 400

42.Based on the graph, what is the limiting factor in the reproductive rate of the female water fleas?
(A) Vapor pressure
(B) Food
(C) Temperature
(D) Density
(E) Competition

43.
Which of the following statements is true concerning the results of the experiment?
(A) The water fleas in the 1-liter jar have a lower reproductive rate than the water fleas in the 0.5-ilter jar.
(B) The reproductive rate of the water fleas in jars A and B are similar.
(C) The reproductive rate in the 0.25-liter jar changes because of a change in habitat.
(D) The water fleas in the 0.25-liter jar are infertile.
(E) The reproductive rate for the water fleas
steadily decreases after 20 days.

44.During a study session about evolution, one of your fellow students remarks, ʺThe giraffe stretched its
neck while reaching for higher leaves; its offspring inherited longer necks as a result.ʺ Which statement is
most likely to be helpful in correcting this studentʹs misconception?

(A) Characteristics acquired during an organismʹs life are generally not passed on
through genes
(B) Spontaneous mutations can result in the appearance of new traits
(C) Only favorable adaptations have survival value
(D) Disuse of an organ may lead to its eventual disappearance
(E) Overproduction of offspring leads to a struggle for survival

45.Homologous structures, which have similar underlying structures but may have different functions, are
formed by
(A) divergent evolution
(B) speciation
(C) segregation
(D) convergent evolution
(E) stabilizing selection

46.All of the following statements are correct about evolution EXCEPT


(A) evolution is influenced by changes in the environment
(B) evolution occurs in populations, not individuals
(C) evolution does not always cause organisms to become more complex
(D) evolution does not occur at the same rate in all organisms
(E) evolution is always a gradual process

52
Classified Real Questions
SAT II Biology E/M Dr Haitham AbdAllah
0100 36 777 19

Questions 47-49 :

A survey reveals that 25 percent of a population of 1,000 individuals have attached earlobes (are homozygous
recessive for the trait). For the following questions, assume that the population
fits the parameters of the Hardy-Weinberg law

47. What is the frequency of the recessive allele?


(A) √0.75
(B) 0.75
(C) √0.50
(D) √0.25
(E) 0.25

47. Unlike most natural populations, this population is best characterized in which of the following ways?
(A) There is genetic equilibrium.
(B) There is gene flow.
(C) There is genetic drift.
(D) Mutations occur.
(E) Mating is non-random.

49. If p equals the frequency of the dominant allele and q equals the frequency of the recessive
allele, which of the following terms represents the frequency of the individuals who show the
dominant phenotype?

(A) p2
(B) q2
(C) 2pq
(D) p2 + 2pq
(E) p2 + 1

50. Cutting off the tails of bulldogs for show purposes does not result in tailessness in the next generation.
Which of the following is the best explanation for this observation?
(A) The tail of the domestic dog is a vestigial structure.
(B) Bulldogs without tails are less successful in attracting mates.
(C) No inheritable change is produced by cutting off the tails
(D) The tail can be judged to have survival value because all adult mammals have tails.
(E) Inheritance of acquired characteristics is possible only for animals less complex than mammals

51. In a small population of animals, only six individuals possess a certain allele. A large storm randomly kills
many individuals of this population, including the SIX individuals. The gene pool of this population has been
affected by
(A) Natural selection.
(B) Speciation.
(C) Variation.
(D) Genetic drift.
(E) Genetic recombination.

53
Classified Real Questions
SAT II Biology E/M Dr Haitham AbdAllah
0100 36 777 19

Questions52-55 :

Brood size, the genetically determined average number of eggs a bird Jays, was studied in a species of starlings
by using the mark-and-recapture method. Starling hatchlings were marked and three months later were
recaptured. The number of those recaptured bearing the original marks was then recorded. The data appear in
the table below.

52. Females whose genotype leads to which of the following brood sizes will have, on the average, the most
number of surviving young?
(A) 8
(B) 7
(C) 6
(D) 5
(E) 3

53. If brood size is a multiple-gene trait, which of the following in LEAST likely to be responsible for the
brood size in starlings?
(A) Convergence.
(B) Genetic drift.
(C) Mutation.
(D) Selection.
(E) Random mating

54. If conditions remain the same as they were at the time the data were collected, succeeding generations
should produce a population. The graph of this population would look like which of the following?

55. If conditions remain the same as they were at the time the data were collected, what should happen to
brood size in starlings over time?

54
Classified Real Questions
SAT II Biology E/M Dr Haitham AbdAllah
0100 36 777 19

(A) It should increase to 10.


(B) It should remain at 4 to 5.
(C) It should decrease to 1 to 2.
(D) Clutches of every size from 1 to 10 should be equally common.
(E) Clutch sizes of 1 to 2 and 7 to 8 should become the most common

56. Which of the following is a characteristics of a four-chambered heart that confers an evolutionary
advantage ?

(A) Blood mixes in the ventricle


(B) If one ventricle fails, the other can compensate
(C) The arteries from the heart are smaller
(D) There is separate circulation to the lungs
(E) The muscle cells in the heart do not need oxygen

57. Although the seal and the penguin both have streamlined. Fish-like bodies with a layer of insulating fat ,
they are not closely related. This similarity results from
(A) convergent evolution
(B) adaptive radiation
(C) homologous evolution
(D) coevolution
(E) parallel evolution

58. Sexually reproducing organisms show greater variation than asexually reproductional ones because
(A) They exhibit fewer mutations
(B) They exhibit a greater mutation rate
(C) Asexually producing organisms do not have internal membranes
(D) Their alleles recombine
(E) They are larger

55
Classified Real Questions
SAT II Biology E/M Dr Haitham AbdAllah
0100 36 777 19

ANSWER KEY ( EVOLUTION ) :

1. C
2. D
3. C
4. C
5. A
6. E
7. B
8. C
9. E
10. C
11. D
12. D
13. E
14. C
15. A
16. C
17. C
18. D
19. B
20. A
21. D
22. D
23. A
24. E
25. D
26. A
27. E
28. B
29. E
30. E
31. C
32. B
33. B
34. B ( a principle of natural selection is that environmental
resources are limited and cannot support an infinitely growing
population. Therefore, individuals with certain phenotypes are more
likely to pass on their alleles (that is, they have greater fitness).

56
Classified Real Questions
SAT II Biology E/M Dr Haitham AbdAllah
0100 36 777 19

Random mating and stable genotype frequencies characterize a


population in Hardy-Weinberg equilibrium, in which natural
selection is not acting. )
35. D (The frequency of heterozygotes in the population can be
determined using the Hardy-Weinberg equation, p2 + 2pq + q2 = 1. The
proportion of heterozygotes equals the middle term, 2pq, where p is the
dominant allele frequency and q is the recessive allele frequency. In this
example, q is given as 0.6 and p must be 0.4 (because p + q = 1). The
heterozygote frequency is therefore 2 × 0.4 × 0.6 = 0.48.

36. D ( Populations can be described as the evolutionary unit because changes in the genetic makeup
of populations can be measured over time. (A), Genetic changes occur only at the individual level. (B),
Only under Hardy-Weinberg equilibrium does the gene pool remain fixed over time in a population.
However, this statement does not explain why the population is the evolving unit. (C), This statement is
true but does not address the question. (E), This statement is also true, but it does not address the question

37.B
38. A
39. D
40. C
41. D ……The graph shows the average number of offspring per female per day. Because there are
100 females in the 1/2 liter, the total number of offspring on the twentieth day would be 100
females times 2 offspring per day, which equals 200 offspring

42. D ….. The fact that lines A and B decrease suggests that the reproductive rate is related to the
population density. After several weeks, as the number of offspring in both jars increase,
the reproductive rate decreases

43. B ….. Based on the graph, the number of Daphnia offspring in jar B is half that of jar A and the
reproductive rate is half. Therefore, the reproductive rate in jar B is the same as the reproductive
rate in jar A

44. A
45. A
46. E
47. D
48. A
49. D
50.C
51. D

57
Classified Real Questions
SAT II Biology E/M Dr Haitham AbdAllah
0100 36 777 19

TAXONOMY

1. Preliminary observations of a new organism reveal the following cellular characteristics: nuclei,
chromosomes, and chloroplasts. Classification would be limited to which of the following
kingdoms?
I. Animalia
II. Plantae
III. Protista
(A) I only.
(B) II only.
(C) III only.
(D) I and II only.
(E) II and III only.

2.
Which of the following is true of archaebacteria (Archaea)?
(A) They are made up of eukaryotic cells
(B) They generally inhabit the most hospitable environments on Earth
(C) They share chromosomal characteristics with Eukarya
(D) They are multicellular
(E) They are much more dominant than they were 3 billion years ago

3. In biological taxonomy, if two animals belong to the same class, they must also belong to the
same.

(A) Family
(B) Genus
(C) Order
(D) Phylum
(E) Species

4. The phylum Arthropoda includes all of the following EXCEPT the


(A) Spider
(B) Lobster
(C) Centipede
(D) Beetle
(E) Leech

5. Fish gills and vertebrate lungs share all of the following features for gas exchange EXCEPT

58
Classified Real Questions
SAT II Biology E/M Dr Haitham AbdAllah
0100 36 777 19

(A) Moist respiratory surface


(B) Large surface area
(C) Proximity of blood vessels
(D) Access to an oxygen source
(E) Countercurrent exchange

6. An organism that has bilateral symmetry, three embryonic layers, deuterostome development, and
a notochord would be classified into which of the following phyla?

(A) Porifera
(B) Chordata.
(C) Echinodermata.
(D) Mollusca
(E) Annelida

7. Comparing which of the following most accurately determines how closely related two species of
plants are?
(A) Fruit color
(B) Branching pattern
(C) Location of flower
(D) DNA nucleotide sequence
(E) Ribosome structure

8. Lichens are an example of


(A) Competition
(B) Mutualism.
(C) Predation.
(D) Parasitism.
(E) Exocytosis

9. The great similarity in appearance between fossil cockroaches and modern cockroaches suggests that
(A) Cockroaches are no longer evolving
(B) The cockroach body plan is highly successful
(C) No other organisms compete with cockroaches
(D) The cockroach food supply has not changed
(E) Only the internal organs of cockroaches can change

10. Since cats and dogs belong to the same class, they must also belong to the same

(A) Genus
(B) Species
(C) Family
(D) Phylum
(E) Order

11. Which of the following organisrm is able to regulate its own body temperature?
(A) Frog
(B) Fish
(C) Snake
(D) Sparrow

59
Classified Real Questions
SAT II Biology E/M Dr Haitham AbdAllah
0100 36 777 19

(E) Turtle

12. A mushroom is most like a


(A) Moss
(B) Fern
(C) Yeast
(D) Pine
(E) Seaweed

Questions 13-14 :

The following data table shows the number of different amino acids in the beta hemoglobin chain of
various organisms compared to the human beta chain

13. which of the following organisms are humans most closely related, based on hemoglobin amino acid
sequence?
(A) Mouse
(B) Monkey
(C) Chicken
(D) Gibbon
(E) Lamprey

14. Human hemoglobin and gorilla hemoglobin is even more closely related than the organisms shown
above; they differ by only a single amino acid. Yet humans and gorillas are considered to be separate
species. This is because
(A) Their hemoglobin chains differ by a single amino acid
(B) Human hemoglobin and gorilla hemoglobin have different functions
(C) They are unable to interbreed
(D) They are the result of convergent evolution
(E) They are the result of parallel evolution

15.Which of the following is the most inclusive taxonomic category?


(A) Order.
(B) Phylum.
(C) Kingdom.
(D) Class.
(E) Species

60
Classified Real Questions
SAT II Biology E/M Dr Haitham AbdAllah
0100 36 777 19

16. The major difference between protostomes and deuterostomes is found in which of the following
characteristics?
(A) Body segmentation
(B) Cephalization
(C) Body symmetry
(D) The origin of the mouth and anus
(E) The type of skeleton present

17. An organism possessing the following characteristics is observed: asexual reproduction, eukryotic,
heterotrophic, chitinous cell walls. This organism most likely belongs to which of the following
taxonomic groups?
(A) Bacteria
(B) Archaea
(C) Fungi
(D) Plantae
(E) Animalia

18. All of the following are mammals EXCEPT


(A) tiger
(B) ape
(C) kangaroo
(D) blue jay
(E) duck-billed platypus

19. Which of the following exhibits internal fertilization, external development of the embryo, few eggs,
and much parenting?
(A) Mammals
(B) Amphibians
(C) Reptiles
(D) Birds
(E) Fish

20. Fish gills and vertebrate lungs share all of the following features for gas exchange EXCEPT
(A) Moist respiratory surface
(B) Large surface area
(C) Proximity of blood vessels
(D) Access to an oxygen source
(E) Countercurrent exchange

Questions21-23 :are based on the chart shown below in which the letters indicate points of
divergence between groups of organisms.

61
Classified Real Questions
SAT II Biology E/M Dr Haitham AbdAllah
0100 36 777 19

Choose from the chart above the letter that best identifies each of the following.
21. Segmentation versus no segmentation.
22. Nails versus claws.
23. Dorsal versus ventral nerve cord

24. Preliminary observations of a new organism reveal the following cellular characteristics: nuclei,
chromosomes, and chloroplasts. Classification would be limited to which of the following kingdoms?
I. Animalia.
II. Plantae.
III. Protista
(A) I only
(B) II only
(C) III only
(D) I and II only
(E) II and III only

25. Which of the following is an advantage that multicellular organisms have over unicellular organisms?

(A)Multicellular organisms lack complete coordination of cells


(B)Multicellular organisms are motile
(C)Multicellular organisms have intercellular communication
(D)Multicellular organisms have cells that are specialized for specific functions
(E)Multicellular organisms reproduces by fusion of an egg and a sperm cell

26. Which of the following kingdoms contains organisms that are autotrophs?
I. Monera
II. Protista
III. Fungi
IV. Plantae
V. Animalia

(A) I , III , IV only


(B) IV only
(C) II and IV only

62
Classified Real Questions
SAT II Biology E/M Dr Haitham AbdAllah
0100 36 777 19

(D) I , II , IV only
(E) I , II , IV and V only

27. Which of the following animal groups is exothermic?


(A) aves
(B) reptilia
(C) carnivora
(D) cetaceans
(E) primates

28. Nematocysts are characteristics of


(A) Porifera
(B) Protozoa
(C) Cnidarians
(D) Annelida
(E) Echinodermata

29. According to the modern theory of evolution, which of the following evolved first?
(A) Krebs cycle
(B) Anaerobic respiration
(C) Autotrophic nutrition
(D) Photosynthesis
(E) Chemosynthesis

30. Which of the following has a chitinous exoskeleton?


(A) Oriole
(B) Starfish
(C) Clam
(D) Honeybee
(E) Earthworm

31. Reptiles
(A) Are homeotherms
(B) Respire with gills
(C) Must live in water at some stage of their life cycle
(D) Possess notochord as adults
(E) Lay leathery eggs

32. A microscopic unicellular organism is observed to have the following characteristics; a food gullet, a
flagellum, chloroplasts, mitochondria, and a nucleus. This organism belongs to which kingdom?
(A) Protista
(B) Plantae
(C) Fungi
(D) Animalia
(E) Monera

33. In evolutionary terms, organisms belonging to which category would be the


most similar?

63
Classified Real Questions
SAT II Biology E/M Dr Haitham AbdAllah
0100 36 777 19

(A) Genus
(B) Family
(C) Order
(D) Kingdom
(E) Class

34. Two species of the same order must be members of the same
(A) Class
(B) Habitat
(C) Genus
(D) Family
(E) Subfamily

35. All of the following are metabolic wastes EXCEPT


(A) Water
(B) Carbon dioxide
(C) Uric acid
(D) Urea
(E) Oxygen

Questions 36-38 :
(A) Phylum Mollusca
(B) Phylum Cnidaria
(C) Phylum Arthropoda
(D) Phylum Annelida
(E) Phylum Echinodermata

36. Members have a foot, radula, and mantle


37. compasses more species than all other animal phyla combined
38. Includes sea urchins, sea cucumbers, and sea stars

Questions 39-41 :
refer to the dichotomous key below.
Taxonomists use dichotomous keys to identify the name of an organism A dichotomous key bas a series
of two-part questions based on the characteriStics of the organisms. When you answer a question, you are
led to the next question and ultimately to the name of the unknown organism. The following is a
dichotomous key for animals in a natural history collection.

Dichotomous Key

1. Has 6 legs go to 2 has more= than 6 legs go to 4


2. Hind legs are greatly enlarged Orthoptera All legs are approximately the same size go to 3

64
Classified Real Questions
SAT II Biology E/M Dr Haitham AbdAllah
0100 36 777 19

3. Has bard wings that are spotted Coleoptera Has soft wings that are striped Lepidoptera
4. Has 8 legs Arachnida Has more than 8 legs Chilopoda

39. Organism #1 is a (n)


(A) Ortboptera
(B) Chilopoda
(C) Lepidoptera
(D) Coleoptera
(E) Arachnids

40. All of the organisms shown are members of phylum


(A) Insecta
(B) Arthropoda
(C) Animalia
(D) Echinodermata
(E) Arachnida

41. Another example of a member of this phylum would be a(an)


(A) Snail
(B) Clam
(C) Earthworm
(D) Lobster
(E) Jellyfish

42.
A student observes an organism under a microscope. It is unicellular, eukaryotic, has threadlike branches,
and a chitinous
cell wall. Which kingdom is the organism most likely to be classified under?
(A) Fungi
(B) Monera
(C) Protista
(D) Animalia
(E) Plantae

43.
Which of the following animals possess nephridia?
(A) Fish
(B) Amphibian
(C) Flatworms
(D) Earthworms
(E) Insects

44. All of the following are true concerning fungi EXCEPT


(A) they reproduce sexually and asexually
(B) they are eukaryotic
(C) they are photosynthetic
(D) they require O2

65
Classified Real Questions
SAT II Biology E/M Dr Haitham AbdAllah
0100 36 777 19

(E) they have a cell wall

Questions 45-48 :

(A) Conjugation
(B) Transduction
(C) Transformation
(D) Plasmid
(E) Spores
45. The transfer of genes between two E. coli joined by a sex pilus
46. A section of bacterial DNA is packaged in a virus and transferred to a new host cell
47. Small rings of bacterial DNA that carry accessory genes
48. A bacteria takes up segments of naked DNA

49.
Which of the following organisms is most closely related to Lynx rufus?
(A) Felis catus
(B) Canis rufus
(C) Canis lupus
(D) Lynx canadensis
(E) Lutra lutra

50.
Which of the following observations support the endosymbiotic theory?
I. Mitochondria and chloroplasts have their own genes
II. Mitochondria and chloroplasts often exist by themselves
III. Mitochondria and chloroplasts have membranes similar to those of prokaryotes
(A) I only
(B) II only
(C) III only
(D) I and III only
(E) II and III only

51. All of the following are kingdoms EXCEPT


(A) Chordata
(B) Monera
(C) Plantae
(D) Fungi
(E) Protista

66
Classified Real Questions
SAT II Biology E/M Dr Haitham AbdAllah
0100 36 777 19

Use the following Table to answer questions 52–55:

(A) Animalia
(B) Fungi
(C) Plantae
(D) Monera
(E) Insufficient data

52. To what kingdom does specimen 1 belong?


53. To what kingdom does specimen 2 belong?
54. To what kingdom does specimen 3 belong?
55. To what kingdom does specimen 4 belong?

Questions 56-59 :

refer to the data below concerning the general animal body plan of five organisms

Note: + indicates a feature presence in an organism.

67
Classified Real Questions
SAT II Biology E/M Dr Haitham AbdAllah
0100 36 777 19

56. The body plan associated with nematodes is Organism


(A) 1
(B) 2
(C) 3
(D) 4
(E) 5

57. The body plan associated with flatworms is Organism


(A) 1
(B) 2
(C) 3
(D) 4
(E) 5

58. Into what phylum should Organism 5 be placed?


(A) Platyhelminthes
(B) Nematoda
(C) Molluska
(D) Annelida
(E) Chordata

59. All of the following organisms exhibit bilateral symmetry EXCEPT


(A) hydra
(B) mollusks
(C) arthropods
(D) earthworms
(E) fish

60. Cells of most Archaea have all of the following characteristics EXCEPT:
(A) circular chromosomes
(B) histones associated with the DNA
(C) peptidoglycans in the cell walls
(D) introns
(E) plasmids

61. Prokaryotes are classified as belonging to two different domains. What are the domains?

(A) Bacteria and Eukarya


(B) Archaea and Monera
(C) Eukarya and Monera
(D) Bacteria and Protista
(E) Bacteria and Archaea

62.
Arthropods can be characterized by all of the following EXCEPT

(A) a hard exoskeleton


(B) a water vascular system

68
Classified Real Questions
SAT II Biology E/M Dr Haitham AbdAllah
0100 36 777 19

(C) jointed appendages


(D) molting
(E) a segmented body

Questions 63-65 :
refer to the graph below, which shows the relationship between body temperature and the temperature of the
surrounding air in four different types of mammals, in a lizard, and in a snake .

63. According to the graph, which animal's body temperature is most constant at air temperatures between
25°C and 35°C?
(A) Cat.
(B) Platypus.
(C) Spiny anteater.
(D) Snake
(E) Lizard.

64. Which of the following can be inferred from the data in the graph?
(A) The cat and the opossum will have the same body temperature at an air temperature of 43°C.
(B) Reptiles and mammals cannot live in the same environment.
(C) Mammals differ in their ability to regulate their body temperatures.
(D) Opossums are poikilothermic (cold-blooded).
(E) The opossum is more closely related to the platypus than it is to the cat.

65. Which is the independent variable in the experiment?


(A) The body temperature of the lizard.
(B) The body temperature of the cat.
(C) The temperature of the air .
(D) The age of each animal.
(E) The body weight of each animal.

66. Which of the following are characteristics of both bacteria and fungi?
(A) Cell wall, DNA, and plasma membrane
(B) Nucleus, organelles, and unicellularity
(C) Plasma membrane, multicellularity, and Golgi apparatus

69
Classified Real Questions
SAT II Biology E/M Dr Haitham AbdAllah
0100 36 777 19

(D) Cell wall, unicellularity, and mitochondria


(E) Nucleus, RNA, and cell wall

Questions 67-69: refer to the following experiment


A student observes three unicellular organisms under a light microscope, and makes these renderings.

67.Which of these cells are protists?


I. Cell X .
II. Cell Y
III. Cell Z
(A) I only
(B) II only
(C) III only
(D) II and III only
(E) I, II , and III

68. Cell X could be of all the following EXCEPT


(A) Bacteria
(B) Prokaryotic
(C) Heterotrofic
(D) Autotrophic
(E) Eukaryotic

69. The cell wall in cell X is composed of


(A) Chitin
(B) Cellulose
(C) Glucose
(D) Glycogen
(E) Peptidoglycan

70
Classified Real Questions
SAT II Biology E/M Dr Haitham AbdAllah
0100 36 777 19

Answer key ( Taxonomy ) :

1. E
2. C
3. D
4. E
5. E
6. B
7. D
8. B
9. B
10. D
11. D
12. C
13. D
14. C
15. C
16. D
17. C
18. D
19. D
20. E
21. A
22. D
23. B
24. E
25. D
26. D
27. B
28. C
29. B
30. D
31. E
32. A
33. A
34. A
35. E
36. A
37. C
38. E
39. D
40. B
41. D
42. A
43. D
44. C
45. A

71
Classified Real Questions
SAT II Biology E/M Dr Haitham AbdAllah
0100 36 777 19

46. B
47. D
48. C
49. D (because Lynx canadensis is of the same genus as Lynx rufus.
Binomial nomenclature uses the genus name, followed by the species name.
Organisms of the same genus are closely related. Answer B draws your attention
because the species names are the same, but Canis rufus is of a different genus than
Lynx rufus, so the two organisms are not as closely related as are Lynx canadensis
and Lynx rufus )
50. D ( Having their own genes and having membranes like those of prokaryotes
suggest that chloroplasts and mitochondria 1) may well once have been independent
organisms, and 2) may have descended from prokaryotes. Statement II is simply not
true. Mitochondria and chloroplasts do not exist by themselves )
51. A
52. C
53. B ( Although both fungi and animals lack photosynthetic activity and have nuclei,
fungi have cell walls made of the carbohydrate chitin, whereas animal cells have no
cell wall )
54. D ( Bacteria (within Monera) and plants both can or do have photosynthetic
activity. Plants have cell walls made of the carbohydrate cellulose, and bacteria have
cell walls that include much carbohydrate in the peptidoglycan layer. However,
plants have nuclei, ruling them out. The prokaryotes of Monera lack nuclei )

55. A

56. C .. The body plan associated with nematodes is Organism 3.


Nematodes-roundworms-are pseudocoelomates. They have a body
cavity, but the cavity is only partially lined with mesodennally-derived
tissue.

57. B .. The body plan associated with flatworms is organism 2. Flatworms


have no body cavity. They are acoelomates

58. E .. Organism 5 should be placed in the phylum chordata. Coelomates


have a body cavity that forms distinct ce!J layers (ectoderm, mesoderm,
and endoderm). The only answer choice with complex processes,
coelmates deuterostomes, and bilateral symmetry is chordate

59. A .. All of the listed organisms exhibit bilateral symmetry except the
hydra. Organisms with bilateral symmetry have two similar halves on
either side of a central plane. Animals with radial symmetry have body
parts arranged around a central line. Animals that exhibit radial
symmetry are hydra, jellyfish, and sea anemones

60. C .. Unlike members of the Bacteria, the Archaea do not have


peptidoglycans in their cell walls. But the Archaea resemble the Bacteria
because both have plasmids and a circular chromosome. On the other
hand, the Archaea also resemble eukaryotes (Domain Eukarya) because
both have histones and introns

72
Classified Real Questions
SAT II Biology E/M Dr Haitham AbdAllah
0100 36 777 19

61. E

62. B

63. A

64. C

65. C

66. A

67. D

68. E

69. E

73
Classified Real Questions
SAT II Biology E/M Dr Haitham AbdAllah
0100 36 777 19

Plants
Questions 1-3 :

(A) Phototropism.
(B) Phototaxis.
(C) Gravitropism (geotropism).
(D) Chemotaxis.
(E) Photoperiodism

1. Is shown by bacteria swimming toward a higher concentration of nutrients.


2. Is shown by oat coleoptiles bending toward a source of light.
3. Is shown by plant roots growing downward

Questions 4-7 :

(A) Mosses.
(B) Ferns.
(C) Gymnosperms.
(D) Angiosperms.
(E) Fungi

4. Organisms that do not carry out photosynthesis.


5. Organisms that produce flowers.
6. Photosynthetic organisms characterized by absence of conducting tissue.
7. Multicellular organisms responsible for recycling nutrients into the soil

8. Increasing the number of stomata on the upper surface of a leaf would most likely.
(A) Increase evaporation of water from the leaf.
(B) Cause more stomata on the under surface of the leaf to open.
(C) Increase the rate of respiration.
(D) Reduce transpiration.
(E) Prevent the leaves from dropping off in the autumn

9. Which of the following plant cell types is dead at maturity, yet functional?
(A) Parenchyma.
(B) Collenchyma.
(C) Phloem.
(D) Xylem.
(E) Companion cell

10. Sugar synthesized in the leaves of a plant is transported to the roots by way of the
(A) Apical meristem.
(B) Vascular cambium.
(C) Pith.
(D) Xylem.
(E) Phloem

Questions 11-12 :

74
Classified Real Questions
SAT II Biology E/M Dr Haitham AbdAllah
0100 36 777 19

Refer to the diagram of the fern life cycle shown below for the following questions :

11. Which arrow in the diagram represents the process of fertilization?


(A) 1
(B) 2
(C) 3
(D)4
(E) 5

36. Based on the information in the diagram, which of the following statements is NOT true?

(A) The found of the new fern plants first emerge rolled up.
(B) There are two copies of each chromosome in the spore.
(C) The gametophyte is haploid.
(D) Gametophytes of ferns are hermaphrodites.
(E) Ferns form spores

37. The rate of water movement up a tall vascular plant is most dependent on which of the following?
(A) Root pressure.
(B) Phloem pressure.
(C) Transpiration pull.
(D) Sucrose translocation.
(E) Rate of photosynthesis

Questions 14- 17 :

(A) Cuticle
(B) Guard cell
(C) Epidermis
(D) Hair
(E) Bristle

14. Controls gas exchange with the environment by responding to short term changes in environmental
conditions
15. Lipid layer that reduces water loss at the surface of green plants
16. Reduces wind flow and heat exchange in mammals

75
Classified Real Questions
SAT II Biology E/M Dr Haitham AbdAllah
0100 36 777 19

17. Cellular layer that separates a vertebrate from its environment

Questions 18-21:

refer to an experiment to test the effects of indoleacetic acid (IAA) on plant growth.
IAA in lanolin was applied to bean cuttings, as indicated in the diagrams below. Apexes were removed in
cuttings II through V. All cuttings were suspended in water and allowed to develop roots. The results are
shown below. (Lanolin does not affect the biological activity of IAA. )

18. Which of the following are the controls for the effect observed with cutting IV?

(A) I and III only.


(B) II and III only.
(C) II and V only.
(D) I, II, and III only.
(E) I, III, and V only.

19. Which of the following most likely accounts for the difference observed between cuttings IV and V?
(A) Not all the apex was removed in cutting V.
(B) Too little lanolin was applied to cutting V.
(C) Diffusion of IAA was slower in cutting IV than in cutting V.
(D) The cuticle and epidermis of the leaf slowed the penetration of IAA into the vascular tissues
of the plant.
(E) The enzymes in the leaf destroy the activity of IAA.
20. The results obtained from cuttings I, Ill, and IV suggest that the apex is the normal source of.
(A) Roots cells.
(B) Lanolin.
(C) IAA.
(D) Water.
(E) Nucleic acids.
21. This experiment provides evidence to support the conclusion that IAA stimulates root growth because
root growth occurred
(A) At the same time in cuttings II and III
(B) Later in cutting I than in cutting V
(C) Only in cutting IV and V
(D) Earlier in cutting IV than in cutting III
(E) In all cuttings

76
Classified Real Questions
SAT II Biology E/M Dr Haitham AbdAllah
0100 36 777 19

Questions 22-23 :

Student grew two different flowering plants under the same conditions and found that the piant~ flowered
at different times of the year. The student determi~ed that both plants had a critical night length of 13
hours. Plant A only flowered when exposed to a night that was longer than the critical value. Plant B only
flowered when exposed to a night that was shorter than the critical value

22. Under the same conditions as before, the student exposed both plants to several days each of a 14-
hour dark period and a 10-hour light period. Which of the following would most likely result from this
light treatment?

(A) Plant A would flower and plant B would not.


(B) Plant B would flower and plant A would not.
(C) Neither plant would flower.
(D) Both plants would flower
(E) Both plants would die before flowering

23. The student found that a flash of light interrupting the dark period prevents flowering. Under which
of the following conditions would plant A still flower?

24. All of the following are associated with the Kreb's cycle EXCPT:
(A) Carbon dioxide is produced from the chemical rearrangements of citric acid
(B) Energy is released and used to form ATP molecules
(C) Light energy in the green yellow wavelengths is absorbed and utilized
(D) Specific enzymes for each reaction are found within the mitochondria
(E) Hydrogen ions and electrons are transferred to the carrier NAD+

Questions 25-27:
refer to the following diagram A cross section of a com leaf at P.M. on a clear sununer day in a well-
irrigated field is shown below

77
Classified Real Questions
SAT II Biology E/M Dr Haitham AbdAllah
0100 36 777 19

25. A difference between the com leaf shown and a leaf from the same plant under drought
conditions is that under drought conditions, the.
(A) Stomata would be more fully open.
(B) Stomata would be mostly closed.
(C) Leafwould be greener.
(D) Leaf would be more rigid.
(E) Mesophyll cells would be swollen

26. The enzymes of the Calvin cycle in this plant are found in which of the following sites?
(A) The cell walls.
(B) The mitochondria of mesophyll cells.
(C) The vascular tissue.
(D) The bundle sheath cells.
(E) The epidermis.

27. In the diagram, which cells communicate with each other by plasmodesmata?
(A) Stomatal guard cell and epidermal cell.
(B) Vascular tissue and bundle sheath cell.
(C) Mesophyll cell and epidermal cell.
(D) Bundle sheath cell and mesophyll cell.
(E) Spongy cell and epidermal cell

Questions 28- 31:

78
Classified Real Questions
SAT II Biology E/M Dr Haitham AbdAllah
0100 36 777 19

28. which part is responsible for protection during root growth?


(A) Root hairs
(B) Zone of cell division
(C) Zone of elongation
(D) Zone of differentiation
(E) Root cap

29. Which part of the root represents the meristematic cells ?


(A) Root hairs
(B) Zone of cell division
(C) Zone of elongation
(D) Zone of differentiation
(E) Root cap

30. The zone at which cells are specialized is


(A) Root hairs
(B) Zone of cell division
(C) Zone if elongation
(D) Zone of differentiation
(E) Root cap

31. Help in anchoring and increase surface area for absorption


(A) Root hairs
(B) Zone of cell division
(C) Zone if elongation
(D) Zone of differentiation
(E) Root cap
Questions 32-35 :

(A) Gymnosperms

79
Classified Real Questions
SAT II Biology E/M Dr Haitham AbdAllah
0100 36 777 19

(B) Angiosperms
(C) Algae
(D) Tracheophytes
(E) Nontracheophytes

32. Plants that do NOT make use of xylem and phloem


33. Plants that often need help in order to reproduce
34. Plant group that includes mosses and liverworts
35. Group composed of unicellular protists

38. When a seed is set in the ground, roots will grow downward while stems will grow upward. This
plant behavior is an example of
(A) hydrotropism
(B) chemotropism
(C) gravitropism
(D) thigmotropism
(E) circadian rhythm

39. Which is vascular tissue associated with phloem?


(A) vessels
(B) meristem
(C) sieve
(D) tracheids
(E) sclerenchmya

40.
The cells of which of the following structures have a triploid
chromosome number?
(A) pollen
(B) fruit
(C) cotyledon
(D) embryo
(E) all cells have the diploid chromosome number

41. Which is CORRECT about monocots?


(A) Vascular bundles in the stem are in a ring.
(B) Their floral parts are usually in 3s.
(C) They usually have taproots.
(D) The veins in the leaves are netlike.
(E) Most common trees, such as maples and oaks, are monocots.

80
Classified Real Questions
SAT II Biology E/M Dr Haitham AbdAllah
0100 36 777 19

42.

Conifers and flowering plants are classified in the same phylum. Which of the following characteristics
do they share?
(A) They are all perennials.
(B) The xylem is dead at maturity.
(C) They are all woody plants.
(D) They both contain seeds.
(E) They both shed their leaves.

43.
Which of the following structures is NOT part of the pistil?
(A) Ovule
(B) Ovary
(C) Style
(D) Anther
(E) Stigma

44.

The plant tissue that gives girth to a plant during each growing season is called the
(A) phloem
(B) tracheid
(C) secondary xylem
(D) lateral meristem
(E) apical meris tem

45.
All of the following are examples of tracheophytes EXCEPT
(A) trees
(B) moss
(C) grass
(D) corn
(E) beans

Questions 44-47 :

(A) Phototropism
(B) Photoperiodism
(C) Thigmotropism
(D) Gravitropism
44. . Plant growth toward a light source
45. When a Venus flytrap snaps shut in response to an insect
46. Growth of a plant in response to the direction of gravity
47. Biological effects caused by changes in day length

48.
Structural components common to plant cell walls but not part of bacterial cell walls include
which of the following?

81
Classified Real Questions
SAT II Biology E/M Dr Haitham AbdAllah
0100 36 777 19

I. Peptidoglycan
II. Cellulose
III. Lignin
(A) I only
(B) II only
(C) I and III only
(D) II and III only
(E) I, II, and III

49. Elongation of cells in shoot tips of plants is promoted by


(A) abscisic acid
(B) carotene
(C) cytokinin
(D) ethylene
(E) auxin

50.
The euglena possesses an organelle that synthesizes
a polysaccharide for storage. This organelle is the
(A) stigma
(B) pyrenoid
(C) eyespot
(D) cytopharynx
(E) oral groove

51.
Leguminous plants such as soy beans and peas exhibit a mutualistic
symbiosis with
(A) Herbivores
(B) Bacteria
(C) Other leguminous plants
(D) Humans
(E) Soils

52. Which of the following is an actively dividing tissue in plants?


(A) Cambium
(B) Xylem
(C) Endodermis
(D) Phloem
(E) Pith

53. Root hairs are extensions of which of the following kinds of cells?
(A) Cortical
(B) Xylem
(C) Phloem
(D) Epidermal
(E) Meristematic

Questions 55-57 refer in the following figures.

82
Classified Real Questions
SAT II Biology E/M Dr Haitham AbdAllah
0100 36 777 19

55. Numbered structures that contain pistils and stamens include which of the following?
(A) 3 only.
(B) 7 only.
(C) 2 and 3 only.
(D) 3 and 4 only.
(E) 7 and 8 only.

56. Which of the four numbered structures represent dicots?


(A) 1 ,2,4, and 7
(B) 1 ,2,5, and 6
(C) 2,3,4, and 6
(D) 2,5,6, and 7
(E) 3,5,6, and 8

57. Which of the following paired structures are the major organs of photosynthesis?
(A) 1 and 4.
(B) 2 and 6.
(C) 3 and 4.
(D) 3 and 5.
(E) 7 and 8.

Questions 58-59

Refer to the diagram of the fern life cycle shown below for the following questions.

83
Classified Real Questions
SAT II Biology E/M Dr Haitham AbdAllah
0100 36 777 19

58. Which arrow in the diagram represents the process of fertilization?


(A) 1
(B) 2
(C) 3
(D) 4
(E) 5

59. Based on the information in the diagram, which of the following statements is NOT true?
(A) The found of the new fern plants first emerge rolled up.
(B) There are two copies of each chromosome in the spore.
(C) The gametophyte is haploid.
(D) Gametophytes of ferns are hermaphrodites
(E) Ferns form spores

Questions 60-64:

Observations of the respiration rate of pea seedlings at different temperatures over time produced the following
data. The rates are given as a percentage of the rate at 25◦C

Temperature 0 hr 1 hr 2 hr 3 hr 4 hr 5 hr 6 hr 7 hr 8 hr
0 ◦C 100% 60% 40% 10% 10% 10% 10% 10% 10%
10◦ C 100% 75% 60% 40% 40% 40% 40% 40% 40%
25 ◦C 100% 100% 100% 100% 100% 100% 100% 100% 100%
30 ◦C 100% 110% 130% 140% 140% 140% 140% 140% 140%
40 ◦C 100% 130% 140% 170% 160% 150% 140% 130% 120%
50 ◦C 100% 98% 96% 94% 70% 45% 30% 15% 0%

60. If the rate at 30°C is plotted against time, a graph of the results would look like which of the
following?

84
Classified Real Questions
SAT II Biology E/M Dr Haitham AbdAllah
0100 36 777 19

61. If a reading were taken at 7.5 hours at 40°C, the rate most likely would be
(A) 110%
(B) 120%
(C) 125%
(D) 130%
(E) 135%

62. The control-temperature in the experiment is


(A) 0 ◦C
(B) l 0 °C
(C) 25°C
(D) 30°C
(E) 50 ◦C

63. According to the data, a pea seedling is most


likely to die in 8 hours at which of the following
temperatures?
(A) 0 ◦C
(B) 10°C
(C) 25°C
(D) 40°C
(E) 50◦C

64. Based on the data, the best explanation for the respiration rate after 3 hours at 0◦C is that
(A) enzymes hydrolyze at low temperatures
(B) low temperatures slow down reaction rates
(C) enzymes are not affected by temperature changes ·
(D) enzymes are used up within 3 hours
(E) enzymes are inactivated by the changes in pH that occur at 0 ◦C

65. All of the following functions are performed by roots EXCEPT:


(A) Storage.
(B) Mineral absorption.
(C) Cell reproduction.
(D) Anchorage.

85
Classified Real Questions
SAT II Biology E/M Dr Haitham AbdAllah
0100 36 777 19

(E) Leaf production.

66. Which of the following occurs in many angiosperms, but NOT in many vertebrates?
(A) Sexual reproduction
(B) Carbohydrate storage
(C) Cellular respiration
(D) Protein synthesis
(E) Asexual reproduction

67. Fertilizers that are applied to soils contain which of the following substances essential for the growth and
development of plants?
(A) Vitamins.
(B) Inorganic compounds.
(C) Carbohydrates.
(D) Proteins.
(E) Lipids

68. Which of the following adaptive features would be found in flowering plants that live in an arid climate?
(A) Vascular tissues
(B) Stomates
(C) Thick cuticles
(D) Multiple leaves
(E) Megaspores

69. Which of the following best accounts for the ability of legumes to grow well in nitrogen-poor soils?
(A) These plants make their own proteins
(B) These plants have a mutualistic relationship with nitrogen-fixing bacteria
(C) These plants are capable of directly converting nitrogen gas into nitrates
(D) These plants do not require nitrogen to make plant proteins
(E) These plants have developed nitrogen-absorbing root hairs

86
Classified Real Questions
SAT II Biology E/M Dr Haitham AbdAllah
0100 36 777 19

Answer Key ( Plants ) :

1. D
2. A
3. C
4. E
5. D
6. A
7. E
8. A
9. D
10. E
11. A
12. B
13. C
14. B
15. A
16. D
17. C
18. D
19. D
20. C
21. D
22. A
23. E
24. C

87
Classified Real Questions
SAT II Biology E/M Dr Haitham AbdAllah
0100 36 777 19

25. B
26. D
27. D
28. E
29. B
30. D
31. A
32. E
33. B
34. E
35. C
36. C
37. C
38. C ( The cotyledon (3n) results from the fusion of one sperm nucleus (n) and two
polar bodies (2n). A seed consists of an embryo (2n), which resulted from fertilization
of the ovum (n) and a sperm nucleus (n) )
39. B ( Monocots have floral parts oriented in 3s and have parallel veins in the
leaves. Dicots usually have taproots, and the veins in their leaves are netlike. Most
trees are dicots. Palm trees are monocots )
40. D
41. D
42. D
43. B
44. B
45. D
46. E
47. C
48. D ( Cellulose and lignin are structural components unique to plant cells. These materials serve
supportive functions. Peptidoglycan, also called murein, is a complex structure consisting of
polysaccharide chains connected by short sequences of amino acids. This substance composes
part of the bacterial cell wall, but it is not present in plant cells )
49. E
50. C
51. B
52. A
53. D
54. D
55. D
56. A
57. B
58. A
59. C
60. A
61. C
62. C
63. E
64. B
65. E

88
Classified Real Questions
SAT II Biology E/M Dr Haitham AbdAllah
0100 36 777 19

66. E
67. B
68. C
69. B

89
Classified Real Questions
SAT II Biology E/M Dr Haitham AbdAllah
0100 36 777 19

Human Body

Questions 1-3:
refer to the following diagram of a human heart

1. Which of the following represents the correct sequence of heart structures that blood would pass
through when traveling from the right atrium to the lungs and then to the left ventricle?
A) 3, 2, 4, 5, 7
B) 6, 2, 1, 4, 5
C) 3, 2, 1, 5, 7
D) 6, 2, 3, 4, 5
E) 6, 2, 4, 5, 7

2. Which structures carry blood rich in oxygen?


A) 2, 3, 4
B) 3, 5, 7
C) 4, 5, 6
D) 1, 5, 7
E) 2, 4, 5

3. Which structure returns blood poor in oxygen to the heart?


(A) 1
(B) 2
(C) 3
(D) 4
(E) 5

4.. The structure found in human males that functions in both urinary and reproductive systems
is the.
(A) Urethra.
(B) Fallopian tube.
(C) Vas deferens.
(D) Epididymis.

90
Classified Real Questions
SAT II Biology E/M Dr Haitham AbdAllah
0100 36 777 19

(E) Seminal vesicle

5. Villi of the small intestine and alveoli of the lung are alike in all of the following ways EXCEPT.
(A) They increase the surface area for exchange of materials.
(B) They have a thin layer of epithelial tissue.
(C) They have extensive blood vessels
(D) They are extremely numerous
(E) They secrete a high volume of enzymes.

6. Which of the following could explain the fact that a woman whose ovulation is normal may be
infertile?
I. Her fallopian tubes are blocked.
II. Large quantities of luteinizing hormone. LH, are released just prior to ovulation.
III. Sperm movement is inhibited by an incompatible pH in the fluids of the vagina or uterus.
(A) I only.
(B) II only.
(C) I and III only.
(D) II and III only.
(E) I, II, and III.

7. The exchange of O2 and CO2 between blood and air in the lungs occurs through the process of
(A) Diffusion
(B) Osmosis
(C) Secretion
(D) Active transport
(E) Pinocytosis

8. One effect of increasing the concentration of antidiuretic hormone (ADH) is to


(A) Reduce the permeability to water in the loop of Henle
(B) Reduce the pem1eability to water of the collecting ducts
(C) Reduce the blood volume.
(D) Increase the volume and output of urine.
(E) Increase the concentration and decrease the volume of urine.

9. Which of the following statements about temperature regulation in human living in hot environments is
correct?
(A) Perspiration insulates the body and prevents heat loss.
(B) Evaporation of perspiration cools the surface of the skin.
(C) Blood is shunted from the head region to the body cavity.
(D) Blood is more likely to travel near the long bones than near the skin.
(E) Goose bumps cool by decreasing the surface area of the skin.

Questions 10-13 :
(A) Growth hormone
(B) Oxytocin
(C) Progesterone
(D) Aldosterone

91
Classified Real Questions
SAT II Biology E/M Dr Haitham AbdAllah
0100 36 777 19

(E) Glucagon
10. . Increases uterine contractions during child birth
11. . Stimulates the release of glucose to the blood
12. . Induces water reabsorption in the kidney
13. . Prepares the uterus for implantation of the fertilized egg

14. Excessive acid in the stomach can result in a hole in the wall of the organ. What is this hole called?
(A) Tumor
(B) Cancer
(C) Boil
(D) Ulcer
(E) Hemorrhoid

Questions 15-19 :

refer to the following diagrams showing the stages of cleavage

15. The first cell to contain the diploid number of chromosomes is


(A)2
(B) 3
(C)4
(D)6
(E) 9

16. Which layer differentiates into the nervous system?


(A)6
(B) 7
(C)8
(D)9
(E) 10
17. Which Of the following, which is a gamete containing stored food?
(A) 1
(B) 2
(C) 3

92
Classified Real Questions
SAT II Biology E/M Dr Haitham AbdAllah
0100 36 777 19

(D) 8
(E) 9

18. A female gamete con taining the monoploid (haploid) number of chromosomes is
(A)2
(B)3
(C)4
(D)5
(E) 8

19. Which is the first cell to undergo mitotic division?


(A) 3
B) 4
(C) 5
(D) 6
(E) 8

20. In general, mammals show an inverse relationship between heart rate and body size. Which of the
following graphs best indicates this relationship?

21. The number of white blood cells in the human circulatory system is most directly affected by the
(A) Presence or absence of infective agents in the blood
(B) Altitude at which the individual lives
(C) Amount of iron in the individual's diet
(D) Amount of hemoglobin manufactured by the body
(E) Sex of the individual

22. Which of the following is NOT a function of the placenta in human beings?
(A) Conveying nutrients from the mother to the fetus.
(B) Conveying CO2 ,H2O, and urea from the fetus to the moilier
(C) Conveying O2 from the mother to the fetus.
(D) Conveying nerve impulses from the mother to the fetus.
(E) Serving as endocrine organ during pregnancy

23. Coffee contains caffeine, which inhibits the secretion of antidiuretic hormone (ADH). After coffee is
consumed, the kidney produces

93
Classified Real Questions
SAT II Biology E/M Dr Haitham AbdAllah
0100 36 777 19

(A) A larger than normal volume of concentrated urine.


(B) A larger the normal volume of dilute urine.
(C) A smaller than normal volume of dilute urine.
(D) Salts to replace those that were eliminated.
(E) Diuretic hormone (DH) to counteract the effect of caffeine.

24. Sexual reproduction is beneficial to species because it


(A) Makes more efficient use of energy
(B) Increases genetic variability
(C) Brings organisms together for reproduction
(D) Keeps the species constant despite environmental changes
(E) Promotes interbreeding of different species

25. The protozoan that causes malaria in humans is


(A) An autotroph
(B) An herbivore
(C) A carnivore
(D) A scavenger
(E) A parasite

26. Tendons connect _____ to _____ ; ligaments connect _____ to


(A) bone to bone; bone to muscle
(B) bone to muscle; bone to bone
(C) bone to bone; muscle to muscle
(D) muscle to muscle; bone to bone
(E) ligaments to bone; tendons to bones

Questions 27-29 :
In an experiment, equal doses of a certain hormone marked with a harmless radioactive label were
injected into several rats. Periodically thereafter, the amolmt of rad.loacuvity in various tissues (Figure
A), and in subcellular locations of the eventual target cells for the hormone (Figure B ) was measured

94
Classified Real Questions
SAT II Biology E/M Dr Haitham AbdAllah
0100 36 777 19

27. Based on the data in figure A, which of the following is a reasonable conclusion?
(A) Uterine cells lack receptors for .this hormone.
(B) The most likely target of this hormone is the uterus.
(C) The lung is the most likely target tissue for this hormone.
(D) The hormone is probably not transported via the blood to the target cells.
(E) The hormone regulates the glucose level within the target cells.

28. Which of the following is the most likely mechanism of action for this hormone?
(A) The hormone binds tightly to the plasma membrane
(B) The hormone moves through the nucleus on its way to the cytoplasm
(C) The hormone binds to a cytoplasmic receptor and is then transported into the nucleus
(D) The hormone enters cells through chloride ion channels
(E) The hormone utilizes active transport to enter the cell

29. The hormone is most likely.


(A) Insulin.
(B) Norepinephrine.
(C) Antidiuretic hormone (ADH).
(D) Estrogen.
(E) lndolacetic acid (auxin)

Questions 30- 33:


Refer to this graph of an impulse passing across a neuron

95
Classified Real Questions
SAT II Biology E/M Dr Haitham AbdAllah
0100 36 777 19

30. The impulse is passing


31. The sodium-potassium pump is responsible for pumping ions across the membrane
32. A steep gradient of sodium and potassium ions exists at the axon membrane
33. An impulse cannot pass

34. Which of the following indicates the correct path of an electrical impulse through a neuron?
(A) Axon → dendrite → cell body
(B) Dendrite → axon → cell body
(C) Cell body → dendrite → axon
(D) Dendrite → cell body → axon
(E) Axon → cell body → dendrite

Questions 35-37 :
(A) Right ventricle
(B) Aorta
(C) Right atrium
(D) Pulmonary artery
(E) Atrioventricular node
35. The chamber where blood returns to the heart
36. Carries deoxygenated blood to the lungs
37. Causes the walls of the ventricles to contract

Questions 38-41 :
(A) Blood plasma.
(B) Platelets.
(C) White blood cells.
(D) Red blood cells.
(E) Fibrinogen.
38. Responsible for transporting minerals between cells
39. Cells that contain hemoglobin and function in oxygen transport
40. Cells that function in the production of antibodies
41. Cell fragments that function in the blood clotting process

42. Osmoregulation is the maintainance of


(A) Three germ layers.
(B) Body symmetry.

96
Classified Real Questions
SAT II Biology E/M Dr Haitham AbdAllah
0100 36 777 19

(C) A balance of water and ions


(D) Reproductive fertility
(E) Extra-embryonic membranes

43. All of these cell types contain the diploid (2N) number of chromosomes EXCEPT
(A) Primary oocyte
(B) Spermatogonium
(C) Spermatid
(D) Zygote
(E) Oogonium

44. Progesterone is primarily secreted by the


(A) primary oocyte
(B) hypothalamus
(C) corpus luteum
(D) anterior pituitary gland
(E) endometrial lining

45. Polar bodies are the products of


(A) meiosis in females
(B) meiosis in males
(C) mitosis in females
(D) mitosis in males
(E) two of the above

46. Alchol acts as a diuretic because it


(A) increases secretion of ADH
(B) inhibits the release of ADH
(C) is not reabsorbed by the tubule cells
(D) increases the rate of glomerular filtration
(E) decreases the rate of glomerular filtration

47. A sperm cell only needs to carry genetic information and swim. Which three cell parts are most likely
to be found in sperm cells?
(A) Mitochondria, chloroplast, flagellum
(B) Nucleus,ribosomes,mitochondria
(C) Nucleus,chloroplast,flagellum
(D) Nucleus, mitochondria, flagellum
(E) Nucleus, mitochondria, cilia

48.Antidiuretic hormone ( ADH ) causes which of the following?


(A) Decreased secretions of urea by the kidney
(B) Decreased absorption of glucose by Bowman’s capsule
(C) Increased reabsorption of salts by the kidneys
(D) Increased secretions of water by the kidneys
(E) Increased reabsorption of water by the collecting duct

97
Classified Real Questions
SAT II Biology E/M Dr Haitham AbdAllah
0100 36 777 19

49.Smooth muscle develops from which of the following germ layers?


(A) endoderm
(B) mesoderm
(C) epiderm
(D) ectoderm
(E) none of the above

50.Oogenesis is the process by which


(A) primary oocytes produce sperm
(B) primary oocytes produce eggs
(C) the egg implants in the uterus
(D) the egg is released from the ovary
(E) starfish regenerate limbs

51.A person with blood type ( AB ) can receive a transfusion from a person with which blood type(s)
?
(A) Type A only
(B) Type A and B
(C) Type B only
(D) Type A, B, and O
(E) Type O only

52. Suppose a vertebrate animal was unable to properly regulate its blood pH. Based on this
information, which of the following brain structures is most likely to be damaged?
(A) thalamus
(B) hypothalamus
(C) medulla
(D) cerebellum
(E) cerebrum

53.
A lizard in the genus Cnemidophorus can reproduce when the female eggs divide mototically
without being fertilized by sperm. This type of reproduction is called
(A) Sexual reproduction
(B) Parthenogenesis
(C) Regeneration
(D) Budding
(E) Hermaphrodism

54.
Epinephrine and norepinephrine are the fight-or-flight hormones that are released by the
(A) Pituitary gland
(B) Thyroid gland
(C) Adrenal glands
(D) Hypothalamus
(E) Pancreas

55.One of the functions of white blood cells is to ingest and destroy harmful agents, such as bacteria, that
find their way into the blood stream. In order to perform this function, you could expect a white blood cell

98
Classified Real Questions
SAT II Biology E/M Dr Haitham AbdAllah
0100 36 777 19

to have a higher than average number of


(A) Ribosomes
(B) Peroxisomes
(C) Chloroplasts
(D) Lysosomes
(E) Chromosomes

56.All of the fol1owing are true statements about gametes EXCEPT:


(A) They are haploid cel1s.
(B) They are produced only in the reproductive structures.
(C) They bring about genetic variation among offspring.
(D) They develop from polar bodies.
(E) They combine to produce cells with the diploid number of
chromosomes.

57.Which of the following systems most directly regulate behavioral responses in


animals?
(A) Excretory and immune systems
(B) Digestive and endocrine systems
(C) Nervous and skeletal systems
(D) Reproductive and nervous systems
(E) Endocrine and nervous systems

58.Which of the following is LEAST likely to result in a release of adrenaline from the adrenal
glands?
(A) walking down a dark, unfamiliar street alone
(B) participating in a highly selective math competition
(C) representing your school at the county track meet
(D) hanging out with your friends after school
(E) being called to your boss’s office when you arrive late to work

Questions 59-61 : refer to the graph below, which shows the changes in electric
impulse.

99
Classified Real Questions
SAT II Biology E/M Dr Haitham AbdAllah
0100 36 777 19

59.Identify where the membrane is pumping sodium and potassium ions to in order to return the
membrane to the resting potential.
(A) A
(B) B
(C) C
(D) A and B
(E) B and C

60. Identify the action potential


(A) A
(B) B
(C) C
(D) A and B
(E) B and C

61.What is the measurement of the membrane potential at rest?


(A) -100mV
(B) -25mV
(C) -50mV
(D) -70mV
(E) 0mV

62.
In non placental mammals, the embryo obtains its food from the
(A) ovary
(B) uterus
(C) oviduct
(D) yolk sac
(E) allantois

100
Classified Real Questions
SAT II Biology E/M Dr Haitham AbdAllah
0100 36 777 19

63.
Which of the following is NOT a direct function of the vertebrate liver?
(A) Synthesis of plasma proteins
(B) Carbohydrate metabolism
(C) Deamination
(D) Reabsorption of water
(E) Detoxification

Questions 64-67 :refer to the following proteins.

(A) Keratin
(B) Hemoglobin
(C) Actin
(D) Insulin
(E) Pepsin
64. This protein requires iron as a cofactor.
65. This protein functions in muscle contraction.
66. This is the principal protein component of skin.
67. This protein functions optimally in high H+ concentrations.

Questions 68-69 :

(A) Antibodies
(B) Antigens
(C) T cells
(D) B cells
(E) Histamines

68. Make capillaries more permeable as part of the inflammatory response


69. Are proteins that are found on the surfaces of invading viruses and bacteria

70. Which of these is a correct representation of the hierarchy of biological organization from
least to most complex?
(A) organelle of a stomach cell, digestive system, large intestine, small intestine, intestinal
tissue, organism
(B) organelle of an intestinal cell, digestive system, small intestine, large intestine,
intestinal tissue, organism
(C) molecule, intestinal cell organelle, intestinal cell, intestinal tissue, digestive system,
organism
(D) molecule, small intestine, large intestine, intestinal tissue, digestive system, organism
(E) molecule, digestive system, digestive cell organelle, small intestine, large intestine,
intestinal cell, organism

71. In human females, the embryo normally implants in the


(A) Ovaries
(B) Fallopian tube
(C) Oviduct
(D) Uterus
(E) Birth canal

101
Classified Real Questions
SAT II Biology E/M Dr Haitham AbdAllah
0100 36 777 19

80. All of the following are normal functions of the liver EXCEPT
(A) breaks down and recycles red blood cells
(B) removes poisons like alcohol from the blood
(C) produces urea
(D) produces bile
(E) contains bacterial symbionts that produce vitamins

Questions 73-75 : ( -ve feedback mechanism) :

Parathyroid hormone (PTH) is secreted from the parathyroid glands. Under normal conditions, PTH and blood
calcium levels are regulated by negative feedback. The normal PTH value is about 100 ng PTH / liter blood,
and normal calcium value is about 100 mg calcium / liter of blood. The results of an experiment with PTH are
shown below.

73. Which of the following happen to the PTH level if the parathyroid glands were removed?
(A) Blood PTH would increase.
(B) Blood PTH would decrease.
(C) Blood PTH would fluctuate regularly.
(D) Blood PTH would be secreted by another endocrine gland.
(E) Blood PTH would no longer be needed.

74. Which of the following would happen to the calcium level if the parathyroid glands were removed?
(A) Blood calcium levels would increase.
(B) Blood calcium levels would decrease.
(C) Blood calcium levels would fluctuate regularly.
(D) Calcium would be secreted into the blood by another gland.
(E) Calcium from the blood would be deposited in teeth.

75. What would most likely happen if blood calcium fell to 50 mg calcium / liter blood due to a low-calcium
diet?
(A) Blood PTH would decrease.
(B) Blood PTH would not change.

102
Classified Real Questions
SAT II Biology E/M Dr Haitham AbdAllah
0100 36 777 19

(C) The body would begin processes that would make blood calcium decrease further.
(D) The body would begin processes that would make blood calcium increase
(E) The body could not regulate calcium properly, so there would be no change in the various processes that
regulate blood calcium

Questions 76-79:

(A) Agglutination
(B) Acclimation
(C) Passive immunity
(D) Allergic reaction
(E) Active immunity

76. Protection of a newborn mammal by antibodies from its mother's milk.


77. Symptoms often caused in humans by the release of histamine.
78. The clumping reaction of anti-B antibodies with type B red blood cells.
79. Long-term resistance to chicken pox acquired after infection with chicken pox virus

80. Which of the following would happen to an individual playing a long game of basketball outdoors on a hot
afternoon?
(A) The production of thyroxin by the thyroid gland would decrease.
(B) The osmotic pressure of the blood would decrease.
(C) The volume of urine produced would increase.
(D) The concentration of urea in the urine would decrease.
(E) The secretion of antidiuretic hormone from the pituitary gland would increase

81. Which of the following is the correct sequence of events in an action potential after stimulation
of a neuron?
(A) K+ moves in; Na+ moves out.
(B) Na+ moves in; K+ moves out.
(C) Na+ moves in; Ca++ moves out.
(D) Na+ moves in; Cl- moves out.
(E) K+ moves in; organic anions move out

82. Which of the following describes an adaptations to arid environment?


(A) A short loop of henle which allows more filtrate to be removed from the blood
(B) A short loop of henle which allows less water to be removed from the filtrate
(C) A long loop of henle which allows more filtrate to be removed from the blood
(D) A long loop of henle which allows more water to be removed from the filtrate
(E) A long loop of henle which allows less filtrate to be removed from the blood

103
Classified Real Questions
SAT II Biology E/M Dr Haitham AbdAllah
0100 36 777 19

83.The picture above represents some stages in the early development of an embryo. In which of the stages
does gastrulation begin?
(A) 1
(B) 2
(C) 3
(D) 4
(E) 5

84.At the neuromuscular junction, which of the following passes from the nerve cell to the muscle cell?
(A) An electrical signal
(B) A chemical signal
(C) A magnetic signal
(D) An axon
(E) A synapse

85.
Which of the following is true of the mammalian cardiovascular system?
(A) All arteries contain oxygenated blood
(B) The left side of the heart is completely separated from the right side
(C) The pulmonary and aortic semilunar valves are two-way valves
(D) Blood goes from the right atrium directly to the left atrium
(E) Pulmonary circulation includes the left ventricle and the aorta

104
Classified Real Questions
SAT II Biology E/M Dr Haitham AbdAllah
0100 36 777 19

86.The graph above shows the oxygen dissociation curves of maternal hemoglobin and fetal hemoglobin. Based on
the graph, it can be concluded that
(A) fetal hemoglobin surrenders O2 more readily than maternal hemoglobin
(B) the dissociation curve of fetal hemoglobin is to the right of maternal hemoglobin
(C) fetal hemoglobin has a higher affinity for O2 than does maternal hemoglobin
(D) fetal and maternal hemoglobin differ in structure
(E) fetal hemoglobin is converted to maternal hemoglobin

87. Hemoglobin is a molecule that binds to both O2 and CO2. There is an allosteric relationship between the
concentrations of O 2 and CO2. Hemoglobin’s affinity for O2
(A) decreases as blood pH decreases
+
(B) increases as H concentration increases
(C) increases in exercising muscle tissue
(D) decreases as CO2 concentration decreases

(E) increases as HCO3 increases

Questions 88-91:

Blood samples are obtained from six different human volunteers. One sample is normal in all its
characteristics. The other samples each have conditions that place one of their blood components outside the
normal range. The samples are spun down in a centrifuge to determine the plasma fractional values. The red
blood cells, white blood cells, and platelets are counted in each of the samples. Finally, the red blood cells are
observed under low partial pressure of oxygen. The data from these observations appear in the table below .

Sample 1 Sample 2 Sample 3 Sample 4 Sample 5 Sample 6

Percent of 52% 53% 55% 56% 57% 53%


blood
volume that
is plasma

Red blood 5.1X106 5.0X106 5.2X106 5.3X106 3.8X106 5.1X106


cells per ml

White blood 7.5x103 18.2X103 7.4X103 7.1X103 7.8X103 7.7X103


cells per ml

Platelets per 4.8X103 5.1X103 5.0X103 5.2X103 5.7X103 0.3X103


ml

Appearance
of red blood
cells under
low partial
pressures of
oxygen

105
Classified Real Questions
SAT II Biology E/M Dr Haitham AbdAllah
0100 36 777 19

88.Which of the following samples is from an individual who is most likely to experience a partial blockage of
the capillary bed?
(A) Sample 1
(B) Sample 2
(C) Sample 3
(D) Sample 4
(E) Sample 5

89.Which of the following samples contain blood from a person whose immune system would be LEAST
effective in fighting infections?
(A) Sample 2
(B) Sample 3
(C) Sample 4
(D) Sample 5
(E) Sampe 6

90.Blood in which of the samples would be less effective than normal blood in carrying oxygen throughout the
body?
(A) Samples 1 only
(B) Samples 3 only
(C) Samples 1 and 5 only
(D) Samples 3 and 5 only
(E) Samples 1,3 and 5

91.blood from the person most likely to have a fever is found in which of the following samples?
(A) Sample 2
(B) Sample 3
(C) Sample 4
(D) Sample 5
(E) Sample 6

106
Classified Real Questions
SAT II Biology E/M Dr Haitham AbdAllah
0100 36 777 19

Answer Key : ( Human body )


1. E
2. D
3. C
4. A
5. E
6. C
7. A
8. E
9. B
10. B
11. E
12. D
13. C
14. D
15. B
16. E
17. B
18. A
19. A
20. C
21. A
22. D
23. B
24. B
25. E
26. B
27. B
28. C
29. D
30. B
31. D
32. A
33. D
34. D
35. C
36. D
37. E
38. A
39. D
40. C
41. B
42. C
43. C
44. C

107
Classified Real Questions
SAT II Biology E/M Dr Haitham AbdAllah
0100 36 777 19

45. A
46. B
47. D
48. E
49. B
50. B
51. D
52. B
53. B
54. C
55. D
56. D
57. E ( The two systems that regulate behavior and how we respond to our environment are the
nervous system (that's our immediate response) and the endocrine system (that's our slow
response). The excretory, reproductive, and digestive systems do not directly regulate any
behavioral responses )
58. D

59. C ( This is called the refractory period and no impulse can pass. )

60. B ( This is where the impulse is passing. The membrane is depolarizing. )

61. D ( The horizontal line represents the membrane at rest or resting potential. It measures -
70Mv )

62. D ( In nonplacental mammals, the embryo obtains its food from the yolk sac. The yolk sac
provides food for the embryo. (A), The ovary is where eggs mature. (B), The uterus is the organ that contains the
developing embryo. (C), The oviduct (also known as the fallopian tube) is a tube that carries the egg from the
ovary toward the uterus. (E), The allantois is an extraembryonic sac that gets rid of wastes )
63. D
64. B
65. C
66. A
67. E
68. E
69. B
70. C
71. D
72. E
73. B
74. B
75. D
76. C
77. D
78. A

108
Classified Real Questions
SAT II Biology E/M Dr Haitham AbdAllah
0100 36 777 19

79. E
80. E
81. B
82. D
83. E
84. B
85. B
86. C
87. A
88. E
89. C
90. C
91. A

109
Classified Real Questions
SAT II Biology E/M Dr Haitham AbdAllah
0100 36 777 19

Ecology

1. An organism that feeds at several trophic levels is


(A) A carnivore
(B) An omnivore
(C) A primary consumer
(D) An herbivore
(E) A primary producer

2. Which of the following represents the proper ecological hierarchy ?


(A) Population → community → ecosystem→ biosphere
(B) Ecosystem → Community → population→ biosphere
(C) Population→ ecosystem → community → biosphere
(D) Biosphere → ecosystem → population → community
(E) Community → population → biosphere → ecosystem

3. . An organism's total interrelationships within its environment constitute its


(A) Trophic level
(B) Dominance rank
(C) Ecological niche
(D) Caste
(E) Habitat

4. What is the characteristic of the biomass at each successive trophic level of an ecosystem?
(A) It increases, due to the reproduction of plants and animals.
(B) lt increases, due to the constant uptake of energy from the sun.
(C) lt remains the same, due to the balance of births and deaths.
(D) It decreases, due to the conversion of food energy to heat energy by organisms.
(E) It decreases, due to the shorter life span of top consumers.

5. Which of the following lists includes only abiotic factors of an ecosystem?


(A) Maples, oaks, and phytoplankton
(B) Cattails, pond lilies, and atmospheric gases
(C) Sunlight, water, and amphibians
(D) Producers, consumers, and decomposers
(E) Water, soil, minerals, and sunlight

6. Which of the following contributes most to the production of acid rain?


(A) Methane (CH4 ).
(B) Sulfur dioxide (SO2).
(C) Ozone (O3).
(D) Carbon monoxide (CO).

110
Classified Real Questions
SAT II Biology E/M Dr Haitham AbdAllah
0100 36 777 19

(E) Nitrogen gas (N2)

7. The stratospheric ozone layer provides protection from which of the following?
(A) UV radiation
(B) Airborne particulate matter.
(C) Chlorofluorocarbons.
(D) Meteorite impact.
(E) Automobile exhaust.

8. Net primary productivity can be expressed as the increase in which of the following in an
ecosystem per unit of time?
(A) Biomass.
(B) Population.
(C) Species diversity.
(D) Biosphere.
(E) Trophic levels

9. According to the concept of Darwinian fitness, the fittest individual is the one who
(A) Lives longest.
(B) Spends the most time in courtship.
(C) Is the strongest individual in its population.
(D) Can escape predators most easily.
(E) Has the most surviving fertile offspring

10. In a food pyramid, which of the following derives the most energy from the amount originally
contained in a kilogram of plant material?
(A) A human drinking cow's milk
(B) A fox eating a rabbit
(C) A cow eating grass
(D) A sea gull eating fish
(E) A mountain lion eating a fox

Questions 11-14 :

(A) Parasitism.
(B) Mutualism.
(C) Commensalism.
(D) Mimicry.
(E) Altruism

11. A tickbird feeds on harmful parasites it pulls from the skin of a rhinoceros.
12. A ground squirrel gives an alarm call, which may make the squirrel's position more
conspicuous to a predator, but
which alerts other grou8nd squirrels to approaching danger.
13. An egret follows a cow, eating the insects that are dispersed as the cow walks.
14. An ant protects and cares for aphids, which in turn produce a secretion that the ant eats .

15. The biome that supplies the largest amount of food for humans is the.
(A) Tundra.

111
Classified Real Questions
SAT II Biology E/M Dr Haitham AbdAllah
0100 36 777 19

(B) Taiga.
(C) Grassland.
(D) Deciduous forest.
(E) Tropical rain forest

16. Fungi can act in ecosystems in which of the following roles?


I. Autotrophs.
II. Parasites.
III. Decomposers.
(A) I only.
(B) II only.
(C) III only.
(D) II and III only.
(E) I, II, and III

17. A skunk that eats rubs, insects, wonns, berries, and roots of plants can be best described as.
(A) A producer.
(B) An herbivore.
(C) A carnivore.
(D) An omnivore.
(E) A decomposer

18. Which of the following roles do maple trees fill in a community?


(A) Primary consumers.
(B) Secondary consumers.
(C) Tertiary consumers.
(D) Decomposers.
(E) Producers

19. Which of the following best describes a climax community?


(A) It does not depend on decomposers for the recycling of nutrients.
(B) It contains more secondary consumers than primary consumers. ·
(C) It consists of a single dominant species thriving in its habitat.
(D) Its species composition remains stable over time.
(E) It does not retain inorganic chemicals

Questions 20-23 : refer to the following diagram of a terrestrial food web, in which the arrows
indicate the direction of energy flow among the species

112
Classified Real Questions
SAT II Biology E/M Dr Haitham AbdAllah
0100 36 777 19

20. Of the following, which species is most likely photosynthetic?


(A) 1
(B) 6
(C) 7
(D) 9
(E) 11

21. Which of the following organisms receives the largest share of the energy contained in
species3?
(A) 4
(B) 6
(C) 7
(D) 10
(E) 11

22. Of the following species, which is most likely a herbivore?


(A) 2
(B) 5
(C) 8
(D) 10
(E) 11

23. When species 9 feeds on species 8, species 9 acts as a


(A) Producer
(B) Decomposer
(C) Omnivore
(D) Predator
(E) Herbivore.
24. Trees, earthworms, fungi, mosses; and insects are examples of which of the folJowing factors
in a forest ecosystem?
(A) Geologic.
(B) Abiotic.
(C) Biotic.
(D) Biogeochemical.
(E) Physical

25. Which of the following is characteristic of mollusk shells?


(A) They are secreted by the mantle.

113
Classified Real Questions
SAT II Biology E/M Dr Haitham AbdAllah
0100 36 777 19

(B) They are secreted by the digestive system.


(C) They are primarily composed of silica.
(D) They are produced by metabolism in foot.
(E) They are found in all mollusks

26. Which of the following would have the LEAST direct effect on the population density of a
species?
(A) Predators.
(B) Competitors.
(C) Decomposers.
(D) Parasites.
(E) Nutrients

27. Fungi are typically found in which of the following ecological roles?
(A) Producers only.
(B) Primary consumers only.
(C) Secondary consumers only.
(D) Both producers and consumers.
(E) Decomposers

28.Which ofthe following is a density-independent factor in population growth?


(A) Predation.
(B) Availability of food.
(C) Disease.
(D) Mating.
(E) Severe weather

29. In a food chain, the concentration of some environmental pollutants increases with trophic
level. For example, in a particular food chain, the DDT concentration in zooplankton is 0.04 ppm,
in small fish it is 0.5 ppm, and in fisheating birds it is 25 ppm. This phenomenon is referred to as
(A) Active concentration.
(B) Active transport.
(C) Bioremediation.
(D) Biomagnification.
(E) Primary production

30. The diagram below represents a biomass pyramid

Which of the following is a true statement concerning the energy in this pyramid?
(A) Decomposers are the main source of energy for this pyramid.
(B) Compared to the other trophic levels, the producers contain the least amount of energy

114
Classified Real Questions
SAT II Biology E/M Dr Haitham AbdAllah
0100 36 777 19

(C) The amount of energy available to tertiary consumers is considerably less than for primary
consumers.
(D) Stored chemical bond energy is greater in tertiary consumers than in primary consumers.
(E) Tertiary consumers contain the greatest amount of energy in the system

Questions 31-34 :
In the following diagram of a food web, arrows indicate the direction of energy flow

31.Which of the following organisms behave as omnivores in the food web?


(A) Hawks.
(B) Sparrows.
(C) Weasels.
(D) Snakes.
(E) Grasshoppers

32.Of the following pairs of organism in the food web, which are involved both in a predator-prey
relationship with each other and in competition with each other for food?
(A) Snakes and mice.
(B) Snakes and sparrows
(C) Plants and grasshoppers.
(D) Hawks and grasshoppers.
(E) Hawks and weasels.

33.All of the following organisms may act as secondary consumers in the food web EXCEPT
(A) Sparrows.
(B) Snakes.
(C) Mice.
(D) Weasels.
(E) Hawks
34.What is the maximum number of trophic levels seen in this food web?
(A) 3
(B) 4
(C) 5
(D) 7
(E) 12

Questions 35-37 :
refer to the following diagram of marine biomes

115
Classified Real Questions
SAT II Biology E/M Dr Haitham AbdAllah
0100 36 777 19

35. Which of the following is the area of least productivity?


(A) Intertidal zone
(B) Oceanic zone
(C) Photic zone
(D) Aphotic zone
(E) Neritic zone

36. Organisms that live in the abyssal zone would need all of the following adaptations EXCEPT
(A) Ability to withstand extreme pressures
(B) Well-developed eyes
(C) Tolerance of cold temperatures
(D) Ability to survive in areas oflow nutrient density
(E) Ability to utilize limited resources

37.Organisms that live in the intertidal zone might have which of the following characteristics?
I. Ability to conduct photosynthesis ·
II. Tolerance of periodic drought
III. Tolerance of a wide range of temperatures
(A) I only
(B) II only
(C) I and III only
(D) II and III only
(E) I , II and III

38. Which of the following is a density-independent factor?


(A) Disease
(B) Famine
(C) Floods
(D) Predation
(E) Increase in toxins in the environment

39. Food chains never consist of more than 4 or 5 trophic levels. The reason for this is
(A) energy is lost along the food chain
(B) there are fewer primary consumers in the world than secondary consumers
(C) producers are always shown on the bottom of any food chain
(D) pioneer organisms compete with consumers

116
Classified Real Questions
SAT II Biology E/M Dr Haitham AbdAllah
0100 36 777 19

(E) all of the above are correct

40. The prairie of North America and the steppe of Russia-both grasslands-are examples of the
same
(A) Biome
(B) Ecosystem
(C) Biosphere
(D) Community
(E) Habitat

41. A boy goes to visit his grandparents. As he drives along, he sees man~ birch, oak, and maple
trees that are losing their leaves. What biome type is the boy visiting?
(A) Tropical rainforest
(B)Desert
(C) Taiga
(D) Temperate deciduous forest
(E) Savanna

42. Which of the following is true of the trophic levels in an energy pyramid?
(A) Only about 10% of the energy from one level is transferred to the next
(B) A given organism can only occupy one trophic level
(C) Consumers store more energy than producers
(D) Detritivores produce the energy for organisms at higher trophic levels
(E) Tertiary-level consumers are more numerous. than producers and are likely to use more
energy

Questions 43-45 :
(A) Carnivores
(B) Decomposers
(C) Herbivores
(D) Producers
(E) Omnivores
43. Bacteria that convert the excrement of cattle into simpler substances
44. Rotifers that eat unicellular green algae
45. Minnows that eat only larvae

46. A major role of certain bacteria in ecosystems is the conversion of.


(A) Nitrogen to ammonia.
(B) Oxygen to ozone.
(C) Sodium to sodium chloride.
(D) Hydrogen to water.
(E) Calcium carbonate to carbon dioxide

47. An organism’s niche is :


(A) The range of temperatures that the organism needs to survive
(B) A full description of the place an organism lives

117
Classified Real Questions
SAT II Biology E/M Dr Haitham AbdAllah
0100 36 777 19

(C) The range of physical and biological conditions in which an organism lives and the
way it obtains what it needs to survive and reproduce
(D) All the physical factors in the organism’s environment
(E) All the biological factors in the organism’s environment

48. The lowest level of environmental complexity that includes living and non-living factors is:
(A) Ecosystem
(B) Biosphere
(C) Biome
(D) Community
(E) Population

49. Which of. the following biomes tends to have the greatest temperature swings?
(A) tropical rain forest
(B) temperate deciduous forest
(C) taiga
(D) desert
(E) ocean

50. Which of the following is the best example of an ecological community ?


(A) All of the pigeons inhabiting a city
(B) A school of trout in the Mississippi river
(C) Tropical rain forests worldwide and all of the organisms that inhabit them
(D) All of the plants, insects, rodents and predators inhabiting a small island
(E) An ant colony

51.
Which of the following statements is true about the energy flow through the food
pyramid?
(A) The most energy is at the top of the pyramid, with the tertiary consumers,
because energy increases at each level
(B) The most energy is at the bottom of food pyramid because some is lost as
it pass from producer to trophic levels
(C) The energy is distnbuted equally at each level of the pyramid; very little
is lost or added
(D) Energy doesn't flow through the food pyramid; each level receives energy
from the sun
(E) It depends on the ecological community; in some, there is more energy at
the top of the food pyramid, and in others, there is more energy at the bottom

52. Which of the following is the best example of a biome?


(A) A rocky cave
(B) A dry riverbed
(C) An expansive deciduous forest
(D) A tern nesting area and the terns that inhabit it
(E) The fish that inhabit the area around a coral reef

53. Which of the following characteristics would most likely be associated with an ocean
organism that feeds exclusively on dead organic matter ?

118
Classified Real Questions
SAT II Biology E/M Dr Haitham AbdAllah
0100 36 777 19

(A) Is exclusively unicellular


(B) Has a silicate shell
(C) Is also autotrophic
(D) Lives in the photic zone
(E) Has poor eyesight

54. The relationship between legumes and nitrogen-fixing bacteria demonstrates


(A) Mutualism
(B) Commensalism
(C) Parasitism
(D) Predation
(E) Convergent evolution

55. Plants may use the ammonia (NH3) released by nitrogen fixing bacteria to synthesize all
of the following EXCEPT
(A) Cytosine
(B) Methionine
(C) Enzymes
(D) Carrier proteins
(E) Cellulose

56. The process by which some bacteria transform ammonia ( NH3) into gaseous N2 in the
atmosphere is called
(A) Transpiration
(B) Nitrogen fixation
(C) Denitrification
(D) Ammonifying
(E) Nitrification

57. Of the following, what best explains how animals obtain the nitrogen and amino acids
they require to survive?

(A) By eating plants that have obtained nitrogen from bacteria or other animals that
have eaten these plants
(B) By ingesting nitrogen-fixing bacteria from soil
(C) Through simple respiration of nitrogen in the atmosphere, absorbed either
through skin or lungs
(D) By providing nitrogen-fixing bacteria a habitat within their own bodies
(E) By drinking water that contains dissolved nitrogen

Questions 58-60 :

refer to the following information and graph.


A marine ecosystem was sampled in order to determine its food chain. The results of the study are
shown below.

119
Classified Real Questions
SAT II Biology E/M Dr Haitham AbdAllah
0100 36 777 19

58. Which of the following organisms in this population are secondary consumers?
(A) Sharks
(B) Mackerels
(C) Herrings
(D) Shrimp
(E) Phytoplanktons

59. Which of the following organisms has the largest biomass in this food chain?
(A) Phytoplanktons
(B) Mackerels
(C) Herrings
(D) Shrimp
(E) Sharks

60. If the herring population is reduced by predation, which of the following is most likely to
occur in this aquatic ecosystem?
(A) The mackerels will be the largest predator in the ecosystem.
(B) The shrimp population will be greatly reduced.
(C) The plankton population will be reduced over the next year.
(D) The shrimp will become extinct.
(E) There will be no change in the number of sharks in the ecosystem.

61. Which of the following organisms serve as decomposers in the ecosystem?


(A) Bacteria and viruses
(B) Fungi and bacteria
(C) Viruses and protists
(D) Fungi and viruses
(E) Bacteria and plants

62. The portion of the earth that is inhabited by life is known as the
(A) ecosystem
(B) biosphere
(C) biome
(D) popula tion
(E) community

Questions 63-66 :
(A) Taiga
(B) Tundra
(C) Grassland

120
Classified Real Questions
SAT II Biology E/M Dr Haitham AbdAllah
0100 36 777 19

(D) Tropical rain forest


(E) Temperate deciduous forest

63. The biome at the highest latitude


64. The biome characterized by harsh winters, short summers, and evergreen trees
65. The biome with producers that are adapted to fires
66. The biome with the greatest species diversity

Questions 67-69 :

67. Which of the following is the correct sequence of animals that


appeared first?
(A) Shark-starfish-dinosour-duck-whale
(B) Starfish-shark-dinosaur-duck-whale
(C) Starfish-dinosaur-shark-duck-whale
(D) Dinosaur-starfish-shark-duck-whale
(E) Duck-starfish-shark-dinosaur-whale

68. Which of the following animals excrete uric acid?


(A) Starfish and dinosaur
(B) Shark and whale
(C) Duck and shark
(D) Dinosaur and shark
(E) Dinosaur and duck

69. Which of following animals represent covergent evolution?


(A) Whale and duck
(B) Shark and duck
(C) Starfish and dinosaur
(D) Shark and whale

121
Classified Real Questions
SAT II Biology E/M Dr Haitham AbdAllah
0100 36 777 19

(E) Starfish and shark

70. Which of these ecosystems accounts for the largest amount of


earth’s nutritional resources?
(A) oceans
(B) tropical rain forest
(C) taiga
(D) grasslands
(E) temperate deciduous forest

71. What surface molecules on a plasma membrane are most


important for cell-to-cell recognition?
(A) phospholipids
(B) glycoproteins
(C) cholesterol
(D) protein channel
(E) single ion channels

72. Which of the following organisms is incorrectly paired with its


trophic level?
(A) cyanobacterium-primary producer
(B) grasshopper-primary consumer
(C) zooplankton-primary producer
(D) eagle-tertiary consumer
(E) fungus-detritivore

73. Which of the following is a detritivore?


(A) Moss
(B) Diatoms
(C) Mushroom
(D) Corn
(E) Fern

Questions 74-79 :

The results of a long-term study of the productivity in six different ecosystems are summarized in the
table below. Ecologists measured both the energy stored per square meter ( primary productivity ) and
the organic matter accumulated ( community production )

122
Classified Real Questions
SAT II Biology E/M Dr Haitham AbdAllah
0100 36 777 19

74. In which community are the producers accumulating energy at the greatest rate?
(A) Alfalfa field.
(B) Young pine forest.
(C) Medium-aged oak-pine forest.
(D) Flowing spring.
(E) Saltwater bay.

75. How much energy (Kcal per square meter) is stored in the young pine forest after accounting for
the energy used by I both producers and consumers in that ecosystem?
(A) 12,220
(B) 4,700
(C) 7,500
(D) 4,600
(E) 2,900

76. Which line on the table describes the arithmetic difference between the energy trapped by
producers and the energy used by both producers and consumers?
(A) Gross primary production.
(B) Autotrophic respiration.
(C) Net primary production.
(D) Net community production.
(E) Heterotrophic respiration.

77. The total matter being produced through photosynthesis is equal to the total matter being consumed by
respiration in which two of the following communities?
(A) Medium-aged oak-pine forest and flowing spring.
(B) Young pine forest and medium-aged oak-pine forest.
(C) Young pine forest and flowing spring.
(D) Alfalfa field and mature rain forest
(E) Mature rain forest and saltwater bay .

78. If productivity is measured as the total amount of carbon fixed before cellular respiration is accounted for,
which ecosystem is most productive?
(A) Alfalfa field.
(B) Young pine forest.
(C) Medium-aged oak-pine forest.
(D) Flowing spring.
(E) Mature rain forest.

79. If climax commununities use as much energy as they produce, which of the following is most likely to be

123
Classified Real Questions
SAT II Biology E/M Dr Haitham AbdAllah
0100 36 777 19

in its climax stage?


(A) Medium-aged oak-pine forest
(B) Alfalfa field.
(C) Saltwater bay.
(D) Young pine forest.
(E) Flowing spring

80. At the end of each geologic time periods, large number of species, genera and even families of animals
disappeared from the fossil record. This observation best exemplifies
(A) Convergent evolution.
(B) Gradualistic evolution.
(C) Genetic drift.
(D) Mass extinction.
(E) Adaptive radiation

Answer Key ( Ecology ) :

1. B
2. A
3. C
4. D
5. E
6. B
7. A
8. A
9. E
10. C
11. B
12. E
13. C
14. B
15. C
16. D
17. D
18. E
19. D
20. A
21. C
22. B
23. D
24. C
25. A
26. C
27. E

124
Classified Real Questions
SAT II Biology E/M Dr Haitham AbdAllah
0100 36 777 19

28. E
29. D
30. C
31. B
32. E
33. C
34. C
35. D
36. B
37. E
38. C
39. A
40. A
41. D
42. A
43. B
44. C
45. A
46. A
47. C
48. A
49. D
50. D
51. B
52. C
53. A
54. A
55. E
56. C
57. A
58. C
59. A
60. C
61. B
62. B
63. B
64. A
65. B
66. D
67. B
68. E
69. D
70. A … The oceans make up the marine biome. It is the largest biome and
provides the greatest proportion of the earth’s oxygen (due to the photosynthesis
of algae and archaea) and nutrition

71. B

125
Classified Real Questions
SAT II Biology E/M Dr Haitham AbdAllah
0100 36 777 19

72. C
73. C
74.

126
Classified Real Questions
SAT II Biology E/M Dr Haitham AbdAllah
0100 36 777 19

EXPERIMENTS

Questions 1-3 :

Diploid species of flowering plant produces flowers with petals that are either white, blue, or lavender. In
addition, the petals show either a tight arrangement or an open arrangement on the receptacle. The gene
for petal color is located on a different pair of chromosomes than the gene for petal arrangement. In an
experiment, plants with lavender flowers and tight petals were allowed to self-pollinate and the numbers
of offspring they produced were as follows :

1. According to the data, which of the following is rue about the self-pollinating parental plants?
(A) They were heterozygous for petal arrangement.
(B) They were homozygous recessive for petal color.
(C) They were homozygous dominant for petal color.
(D) They were homozygous for petal arrangement.
(E) They exhibited incomplete dominance for petal arrangement

2. On the basis of the data, it can be concluded that the trait for this plant species exhibits which of the
following inheritance patterns?
(A) Sex-linkage.
(B) Crossing-over.
(C) Incomplete dominance.
(D) Complete dominance.
(E) Epistasis

3. If one of the offspring with white flowers and a tight petal arrangement were allowed to self-pollinate,
its offspring would most likely have.
(A) 100% white flowers. .
(B) 100% lavender flowers.
(C) 50% white flowers.
(D) 50% lavender flowers.
(E) 50% tight petals

Questions 4-6 :

A company wan ted to develop a method to release silver from the gelatin layer on the back: of unused,
out-of-date photographic film. The company tested the effectiveness of extracts of various fruits in the
enzymatic digestion of gelatin, a protein, at various pHs.. The data obtained are shown in the graph below

127
Classified Real Questions
SAT II Biology E/M Dr Haitham AbdAllah
0100 36 777 19

4. According to the data, at which pH does pineapple extract exhibit 50 percent of its rna.xi.o;tal activity?
(A) 2.5 and 6.5
(B) 4.0and·5.7
(C) 5.0 and 5.7
(D) 4.0 only
(E) 6.0 only

5. According to the data, which of the following fruit extracts digests gelatin over the broadest range of
pH?
(A) Pineapple
(B) Grapefruit.
(C) Apple.
(D) Peach.
(E) Banana.

6. According to the data, which of the following fruit extracts requires a neutral mediwn for its maximum
gelatin- digesting activity?
(A) Pineapple.
(B) Grapefruit.
(C) Apple.
(D) Peach.
(E) Banana

Questions 7-9 :refer to the graph below, in which the sold line represents the reaction of an enzyme with
its substrate under normal conditions. Chemical I is added at point I, and the resulting reaction is shown
by a dashed line. Chemical II is added at point II, and the resulting reaction is shown by a dotted line. In
both cases, the same products are formed

128
Classified Real Questions
SAT II Biology E/M Dr Haitham AbdAllah
0100 36 777 19

7. If the reactions between the enzyme and the substrate were allowed to continue until the substrate is no
longer available, a graph depleting the results would look like which of the following?

8.Which of the following would produce the greatest quantity of products after 40 seconds of reaction
with the substrate?
(A) Enzyme only
(8) Chemical II only
(C) Enzyme and chemical I only
(D) Enzyme and chemical II only
(E) Enzyme, chemical I, and chemical II

9. The results of the experiment suggest that chemical II is


(A) An activator
(B) A coenzyme
(C) Another enzyme
(D) Another substrate
(E) An inhibitor

Questions 10-12:

An ecologist was studying two species of cliff-nesting gulls, red-legged and black-legged kittiwakes in
Alaska. One experiment compared the use of artificial nesting, which differed in width and elevation on
the cliff face the results are shown in the figure below

10. From these observations, it appears that.


(A) Black-legged kittiwakes prefer narrow ledges high up on cliffs.
(B) Black-legged kittiwakes prefer wide ledges high up on cliffs.
(C) Red-legged kittiwakes prefer narrow ledges high up on cliffs.
(D) Both species prefer wide ledges high up on cliffs
(E) Neither species shows any preference for width of ledges

129
Classified Real Questions
SAT II Biology E/M Dr Haitham AbdAllah
0100 36 777 19

11. These observed nesting patterns represent which of the following types of ecological interaction?
(A) Predator/prey.
(B) Host/parasite.
(C) Competitive displacement.
(D) Ecological succession.
(E) Mutualism.

12. In another experiment the ecologist planned to put out only intermediate-width ledges. Which of the
following would be a likely outcome from this planned experiment?

(A) Only the larger black-legged kittiwakes would use the intermediate-width ledges.
(B) Only the smaller red-legged kittiwakes would use the intermediate-width ledges.
(C) Neither species would use the intermediate-width ledges
(D) Red kittiwakes would use more of the higher ledges and black kittiwakes would use more of
the lower ledges
(E) Red kittiwakes would use more of the lower ledges and black kittiwakes would use more of
the higher ledges

13. Both species of gull prefer to nest in large groups. All of the following are advantages of this colonial
behavior EXCEPT
(A) Increased chance of finding a mate
(B) Increased resistance to disease
(C) Increased reproductive success
(D) Decreased chance of predation
(E) Exclusion of other gull species

Questions 14-16:

In the laboratory, students tested the ability of an antibiotic to kill a pathogen. Five Petri dishes containing
nutrient and bacterial colonies were set up as follows:
Dish 1: E. coli plated at dilution 1.
Dish 2: E. coli plated at dilution 1.
Dish 3: E. coli plated at a concentration that is 1/2 of that in dish set 2
Dish 4 : Nutrient agar only (no E. coli)
Dish 5: A different strain of E. coli.
Students placed a paper disk soaked in an antibiotic in the center of dishes 2 through 5 using sterile
technique and sterile paper disk on dish 1. They observed the cultures after 24 hours

14. Which of the Petri dishes in the experiment served as the control ?
(A) 1
(B) 2
(C) 3
(D) 4
(E) 5

15. There was no antibiotic action observed in Petri fish 5. This observation is most 1ikely due to which
of the following?
(A) The antibiotic was overdiluted
(B) The agar had more nutrients

130
Classified Real Questions
SAT II Biology E/M Dr Haitham AbdAllah
0100 36 777 19

(C) A different antibiotic was used


(D) The E. coli strain was resistant to the antibiotic
(E) Another microbe species had been

16. Antibiotics are used to inhibit the growth of bacteria in humans because, in most instances, which of
the following is true?
(A) The antibiotics interfere with the replication or translation of prokaryotic, but not eukaryotic,
cells.
(B) The antibiotics alter replication or translation in all cells.
(C) The antibiotics help the human host to metabolize bacteria faster.
(D) The antibiotics prevent the viral infections that often precede bacterial infections.
(E) Bacteria, but not human cells, are able to ingest antibiotics

Questions 17-19 :

Students exploring plant responses to physical stimuli set up three experiments.


experiment l Four seeds were placed on moist paper in a petri dish placed on its edge in the dark. An
illustration of the seeds on the first day and the fifth day is shown in Figure 1.

Experiment II. A vertical stake was placed in the pot with a young pea plant in a lighted area. A second
potted pea plant without a stake was placed next to the first. An illustration of the two plants after five
days is shown in Figure II.

Experiment III. A 6-inch onion plant was placed in a box on a table with only one vertical side open to a
lighted room.
second 6-inch onion plant was placed near the box on the same table. The light was on twenty four hours
a day. An illustration of the plants at the end of five days is shown in Figure III

17. Experiment I demonstrates a response to


(A) Light.
(B) Touch.
(C) Gravity.
(D) PH.
(E) Temperature

131
Classified Real Questions
SAT II Biology E/M Dr Haitham AbdAllah
0100 36 777 19

18. Experiment II demonstrates a response to.


(A) Light
(B) Touch
(C) Temperature
(D) PH
(E) Water

19. The growth responses shown in all three experiments are controlled at the cellular level by

(A) Electrical impulses


(B) Salts
(C) Oxygen
(D) Plant hormones
(E) Transpiration

Questions 20-22 :

Protein stains, such as blood, are difficult to remove from clothes. In order to test the ability of certain
substance to remove protein stains, the following experiment was performed. Fourteen equal-sized pieces
of cotton cloth were each stained with 1 milliliter (mL) of blood. Each piece was then placed in a 1-liter
flask containing 500 mL of H2O and 10 mL of one of the solutions listed in the table below. Each flask
was well shaken during washing at a particular temperature and the cloth was removed at the time shown.
Extent of stain removal under the various conditions is shown in the table below

20. According to the data, the protein-digesting enzyme was most effective at which of the following
temperatures?
(A) 5°C
(B) l0°C
(C) 40°C
(D) 60°C
(E) 70°C

132
Classified Real Questions
SAT II Biology E/M Dr Haitham AbdAllah
0100 36 777 19

21. Protein-stain removal is shown to be time dependent in which of the following pairs of flasks?
(A) 1 and 3
(B) 2 and 3
(C) 3 and 4
(D) 7 and 8
(E) 11 and 12

22. The data show that blood proteins could be removed from the cloth by which of the following?
I. Detergent X
II. Detergent Y
III. Amylase

(A) I only.
(B) II only.
(C) III only.
(D) I and II only.
(E) I, II, and III

Questions 23-27:

Cytochrome C is a respiratory enzyme located in the mitochondria of cells. The amino acid sequence of
cytochrome C was obtained from different species; the differences in the sequences are illustrated below.
The number indicates how many amino acids in the cytochrome C of a particular species differ from
those of various other species

133
Classified Real Questions
SAT II Biology E/M Dr Haitham AbdAllah
0100 36 777 19

23. Based on their differences in cytochrome C , which of the following pairs of organisms are most
closely related to each other?
(A) Rattlesnake and Kangaroo
(B) Monkey and chicken
(C) Turtle and tuna fish
(D) Duck and Rabbit
(E) Yeast and bread mold

24. Which of the following organisms appears to be LEAST closely related to humans?
(A) Kangaroo
(B) Tunafish
(C) Bread mold
(D) Moth
(E) Turtle

25. The DNA that codes for cytochrome C in monkeys is most similar to that in a
(A) Dog
(B) Human
(C) Horse
(D) Chicken
(E) Yeast

26. The data reported in this table demonstrates that cytochrome C is present in
(A) Vertebrates only
(B) Mammals and insects only
(C) Animal and fungi
(D) Bacteria and viruses
(E) Eukaryotes and prokaryotes

27. Which of the following best explains why there are relatively few differences in the cytochrome C of
the diverse organisms in the chart?
(A) Most proteins are the same among theses groups
(B) DNA replication is error-free
(C) All organisms undergo protein synthesis
(D) The respiratory reactions are recent evolutionary occurrences
(E) The respiratory reactions are similar in all eukaryotes

134
Classified Real Questions
SAT II Biology E/M Dr Haitham AbdAllah
0100 36 777 19

Questions 28-30 :

The diagram below shows a seed germination experiment. The lime water in the bottles react with any
carbon dioxide to produce a cloudy solution. At the beginning of the experiment, germinating seeds are
placed in beaker III, and throughout the experiment, air is continuously pumped in at X and flows out at
Y

28. Bottle I was included in this experiment to modify the air available to the germinating seeds by
(A) Increasing nitrogen concentration
(B) Increasing oxygen concentration
(C) Removing carbon dioxide
(D) Removing dust
(E) Removing bacteria

29. What should happen to the liquid in bottle IV during the course of the experiment?
(A) It should remain clear throughout the experiment
(B) It should steadily decrease in temperature during the experiment
(C) It should be clear at the begining of the experiment and then turn cloudy
(D) It should be cloudy at the begining of the experiment and then turn clear
(E) It is impossible to predict from the information given

30. This experiment was designed to determine whether germinating seeds


(A) Need lime water
(B) Produce carbon dioxide
(C) Produce oxygen
(D) Use water vapor
(E) Use oxygen

Questions 31-33 :

Scientists have made a biochemical comparison of five types of proteins found in both humans and
chimpanzees. Their findings are summarized in the following table :

135
Classified Real Questions
SAT II Biology E/M Dr Haitham AbdAllah
0100 36 777 19

31. According to this table, humans and chimpanzees


(A) Have equal amounts of all the proteins
(B) Have equal amounts of the amino acids
(C) Have equal amounts of serum albumin
(D) Are most similar in their transferrin
(E) Are most similar in their cytochrome c

32. The largest protein in the list is


(A) Hemoglobin
(B) Myoglobin
(C) Cytochrome c
(D) Serum albumin
(E) Transferrin

33. The results for cytochrome C lead to the conclusion that


(A) Neither humans nor chimpanzees need it in their respiratory chain
(B) Both humans and chimpanzees differ in their amino acids for this protein
(C) Humans and chimpanzees are closely related in their evolutionary origin
(D) Fossils for humans and chimpanzees have identical DNA .
(E) Humans and chimpanzees have identical hemoglobin

Questions 34-36 :

Five beakers are used in an experiment about osmosis. Each beaker contains 50 mL of a sucrose solution
of varying concenttations: 0.2 M, 0.4 M, 0.6 M, 0.8 M, or 1.0 M. Pieces of fresh potato (each 10.01 in
mass) are cut, weighed, and placed into the beakers. After 12 hours, the potatoes are carefully removed
from each beaker and weighed again

See the data in the table below.

136
Classified Real Questions
SAT II Biology E/M Dr Haitham AbdAllah
0100 36 777 19

34. In this experiment

(A) Water flowed into the potato only


(B) Water flowed out of the potato only
(C) Sucrose flowed into the potato only
(D) Sucrose flowed both into and out of the potato
(E) Water flowed both into and out of the potato

35. What is the molarity (concentration) within the potato cells?


(A) Less than 0.2 M
(B) Less than 0.4 M but greater tban 0.2 M
(C) Less than 0.6 M but greater than 0.4 M
(D) Less than 0.8 M but greater than 0.6 M
(E) Greater than 0.8 M

36. The results of this experiment live support to the theory that
(A) Water diffuses down a gradient
(B) Water can be actively transported agianst a gradient
(C) Solutes will diffuse from high concentration to low concentration
(D) Living cells respond in different ways to the same conditions
(E) Potato cells respond differently from other 1iving cells

Questions 37-40:

Intact chloroplasts are isolated from dark green leaves by low-speed centrifugation and are
suspended into two tubes, A and B, containing cold buffer. A blue dye, DPIP, which turns clear
when reduced, is added to tube A.

137
Classified Real Questions
SAT II Biology E/M Dr Haitham AbdAllah
0100 36 777 19

An experiment is set up to study the effect of different wavelengths of light on the reduction of
chloroplasts and the rate of photosynthesis. Although the wavelengths of light vary, the light
intensity is the same. Here is a graph of the collected data.

37. Which statement best describes the results of the experiment?


(A) The lower the wavelength of light, the greater the rate of photosynthesis
(B) The highest wavelength of light provides the greatest rate of photosynthesis
(C) The highest rate of photosynthesis results from the exposure to two different
wavelengths of light.
(D) The data vary too much to determine a clear relationship between wavelengths of
light and the rate of photosynthesis.
(E) The data are clear. However, the sample is too small and more data are required
to make an accurate determination.

38. What instrument would be best to quantify the color change of the DPIP?
(A) a light microscope
(B) the naked eye is adequate
(C) a spectrophotometer
(D) an electron microscope

138
Classified Real Questions
SAT II Biology E/M Dr Haitham AbdAllah
0100 36 777 19

(E) a magnifying glass

39. The greatest reduction of the DPIP by two different wavelengths of light suggests that
(A) plants absorb various wavelengths of green light
(B) UV light is responsible for the greatest amount of photosynthesis
(C) different portions of the plant absorb different wavelengths of light
(D) the plant contains two separate photosynthetic pigments that absorb light in
different wavelengths
(E) the light shining on the plant contains the entire spectrum of light

40. Which is the dependent variable?


(A) time
(B) both time and the different wavelengths of light
(C) the change in color of the DPIP
(D) the different wavelengths of light only
(E) temperature

Questions 41-42 :

In a study of the development of frogs, groups of cells in the germ layers of several embryos in the early
gastrula stage were stained with five different dyes that do not harm living tissue.
After organogenesis (organ formation), the location of the dyes was noted, as shown in the table below.

41. Ectoderm would eventually give rise to tissues containing which of the following colors?
(A) Red and blue
(B) Yellow and purple
(C) Green and red
(D) Green and yellow
(E) Purple and green

42. Tissues stained with the purple dye were probably derived from
(A) the ectoderm only
(B) the mesoderm only
(C) the endoderm only
(D) both the ectoderm and the mesoderm
(E) both the endoderm and the mesoderm

Questions 43-44 refer to the birth of a child with blood type A to a mother with blood type B

43. The father must have which of the following blood types?
(A) AB only
(B) Either AB or B
(C) Either AB or O

139
Classified Real Questions
SAT II Biology E/M Dr Haitham AbdAllah
0100 36 777 19

(D) Either AB or A
(E) AB or A or O

44.If the father has blood type AB, which of the following statements is correct about the mother?
(A) She contributes an IB allele, which is recessive to the father's IA allele
(B) She contributes an i allele, which is recessive to the father's IA allele.
(C) She contributes an IB allele, which is codominant to the father's IA allele.
(D) She contributes an i allele, which is codominant to the father's IB allele.
(E) She is homozygous for the IB allele.
Section I
Questions 45-48 refer to the information and table below.
The following information was gathered from a study to determine the relationship between structure and
function in the digestive tracts of several animals.

45.Which of the following can be correctly concluded about the relationship between intestinal length and
body size for the organisms in the table?
(A) Carnivores have greater intestinal length relative to body size than do herbivores
(B) Herbivores have greater intestinal length relative to body size than do carnivores
(C) Omnivores have greater intestinal length relative to body size than do either carnivores
or herbivores.
(D) The smaller the intestinal length relative to body size, the more nutrients are absorbed.
(E) No relationship exists between intestinal length and mode of nutrition

140
Classified Real Questions
SAT II Biology E/M Dr Haitham AbdAllah
0100 36 777 19

46.Which of the animals is most likely to have, living in specialized areas of its digestive tract, abundant
symbiotic bacteria that break down foods that the animal normally could not digest on its own?
(A) 1
(B) 2
(C) 3
(D) 4
(E) 5
Section I
47. Which animal has a gastrovascular cavity?
(A) 1
(B) 2
(C) 3
(D) 4
(E) 5

48. Which animal is most efficient in absorbing the food it consumes?


(A) 1
(B) 2
(C) 3
(D) 4
(E) 5

Questions 49-50:

Observations of the respiration rate of pea seedlings at different temperatures over time produced the following
data. The rates are given as a percentage of the rate at 25◦C

Temperature 0 hr 1 hr 2 hr 3 hr 4 hr 5 hr 6 hr 7 hr 8 hr
0 ◦C 100% 60% 40% 10% 10% 10% 10% 10% 10%
10◦ C 100% 75% 60% 40% 40% 40% 40% 40% 40%
25 ◦C 100% 100% 100% 100% 100% 100% 100% 100% 100%
30 ◦C 100% 110% 130% 140% 140% 140% 140% 140% 140%
40 ◦C 100% 130% 140% 170% 160% 150% 140% 130% 120%
50 ◦C 100% 98% 96% 94% 70% 45% 30% 15% 0%

49. If the rate at 30°C is plotted against time, a graph of the results would look like which of the
following?

141
Classified Real Questions
SAT II Biology E/M Dr Haitham AbdAllah
0100 36 777 19

50. If a reading were taken at 7.5 hours at 40°C, the rate most likely would be
(A) 110%
(B) 120%
(C) 125%
(D) 130%
(E) 135%

51. The control-temperature in the experiment is


(A) 0 ◦C
(B) l 0 °C
(C) 25°C
(D) 30°C
(E) 50 ◦C

52. According to the data, a pea seedling is most likely to die in 8 hours at which of the following
temperatures?
(A) 0 ◦C
(B) 10°C
(C) 25°C
(D) 40°C
(E) 50◦C

53. Based on the data, the best explanation for the respiration rate after 3 hours at 0◦C is that
(A) enzymes hydrolyze at low temperatures
(B) low temperatures slow down reaction rates
(C) enzymes are not affected by temperature changes ·
(D) enzymes are used up within 3 hours
(E) enzymes are inactivated by the changes in pH that occur at 0 ◦C

Questions 54-57 refer to experiment described below.

A sample of 0.1 milliliter of an Escherichia coli (E. coli) culture was added to each of 15 petri plates
containing nutrient agar. Then 0.1 milliliter of sterile water was added to each of three plates, and 0.1 milliliter
of either solution I, TI, III, or IV was added to the remaining plates as shown in the table below. The liquid

142
Classified Real Questions
SAT II Biology E/M Dr Haitham AbdAllah
0100 36 777 19

was spread evenly over the agar, and the plates were placed in an incubator at 37°C. After a 48-hour period,
the number of colonies per plate was counted

54. Which of the following hypotheses was tested in this experiment?


(A) Some of the solutions will cause E. coli cells to mutate.
(B) E. coli grows in petri plates as well as in humans.
(C) Water is required for the growth of E. coli
(D) Some of the solutions will inhibit the growth of E. coli.
(E) E. coli grows best at 37°C.

55. The toxicity of solution IV can best be determined by.


(A) Comparing the average number of colonies grown on the solution IV plates with the average number
grown on the solution II plates.
(B) Comparing the average number of colonies grown on the solution IV plates with the average number
grown on the plates to which only water was added.
(C) Comparing the number of colonies on the solution IV plates across trials 1, 2, and 3.
(D) Measuring the overall size of the colonies on the solution IV plates.
(E) Determining the average distance between neighboring colonies on the solution N plates.

56. On the basis of the experimental results, which solution might be most effective as a disinfectant against E.
coli?
(A) Solution I.
(B) Solution II.
(C) Solution III.
(D) Solution IV.
(E) Water.

57. Which of the following is a valid interpretation of the results of the experiment?
(A) Growth on solution II plates is comparable to growth on the control plates.
(B) All solutions stimulate an increase in the rate of bacterial growth.
(C) E. coli cells are not affected by solution IV.
(D) Solution III dissolves the cell wall of bacterial cells.
(E) Solution I is one-half the concentration of solution II.

143
Classified Real Questions
SAT II Biology E/M Dr Haitham AbdAllah
0100 36 777 19

Questions 58-60:

Two hormones that control metamorphosis in the silkworm moth were measured in the four life stages of the
moth. The two hormones are molting hormone ( MH ) and juvenile hormone ( JH ); and the four stages are
egg, larva, pupa, and adult. The figure below shows the relative concentrations of the two hormones during the
four life stages.

58. The metamorphosis of a pupa to an adult moth is correlated with


(A) The absence of molting hormone
(B) A buildup of juvenile hormone
(C) A sharp increase in molting hormone
(D) Equal amounts of molting juvenile hormone
(E) A buildup of both molting and juvenile hormone

59. According to the observed pattern, a constant high level of juvenile hormone would most likely
(A) Maintain the larval stage
(B) Speed up hatching of the adult
(C) Stimulate egg laying.
(D) Cause the larva to pupate
(E) Encourage mating

60. An insecticide that blocked production of molting at the pupal stage would most likely
(A) Increase the number of infertile moths.
(B) Cause the pupae to mature too early.
(C) Increase the number of larvae.
(D) Prevent the larvae from getting larger.
(E) Stop the development at the pupal stage

144
Classified Real Questions
SAT II Biology E/M Dr Haitham AbdAllah
0100 36 777 19

Questions 61-62 :

Crickets recognize potential mates by the frequency at which sound pulses are emitted by other crickets while
chirping. For each species, the number of pulses per second also varies with the temperature, as shown in the
graph below.

61. About how many pulses per second do crickets of species III produce at 25°C?
(A) 10
(B) 25
(C) 35
(D)50
(E)15

62.In the laboratory the species I and II can be induced to interbreed. This attempt is most likely to be
successful at
(A) 0◦C
(B) 15◦C
(C) 25◦C
(D) 35◦C
(E) 75◦C

145
Classified Real Questions
SAT II Biology E/M Dr Haitham AbdAllah
0100 36 777 19

Answer Key ( Experiments ) :

1. D
2. C
3. A
4. B
5. E
6. D
7. B
8. C
9. E
10. C
11. C
12. D
13. B
14. A
15. D
16. A
17. C
18. B
19. D
20. C
21. E
22. B
23. D
24. C
25. B
26. C
27. E
28. C
29. C
30. B
31. E
32. E
33. C
34. E
35. C
36. A
37. C …The lower the line on the graph, the less blue is the DPIP The less DPIP
indicates that more DPIP has been reduced, which means that the rate of
photosynthesis is the fastest

146
Classified Real Questions
SAT II Biology E/M Dr Haitham AbdAllah
0100 36 777 19

38. C …A spectrophotometer measures how much light passes through or is


absorbed by a solution, depending on how you set up the instrument
39. D…. Plants use chlorophyll a plus antenna pigments to broaden the
wavelengths of light that a plant can use for photosynthesis. In addition,
plants have two photosystems, I and II, which absorb light in different
spectra. That is why combined wavelengths of light were more effective in this
experiment
40. C….The dependent variable is the one that changes in an experiment. In this
case, the color of DPIP changes in response to light
41. A
42. C
43. D
44. B
45. B
46. D
47. A
48. D
49. A
50. C
51. C
52. E
53. B
54. D
55. B
56. C
57. A
58. C
59. A
60. E
61. C
62. B

147
Classified Real Questions
SAT II Biology E/M Dr Haitham AbdAllah
0100 36 777 19

Animal
Behaviour

1. Ritualized contests between individuals in a population with little risk of serious injury or death to
participants within the species lead to
(A) A stable dominane hierarchy
(B) Biological altruism
(C) Divergent evolution
(D) Instinctive behavior
(E) A broader habitat

2. A baby goose hatches in captivity and follows its human handler around wherever he goes. This is an
example of
(A) imprinting
(B) conditioning
(C) habituation
(D) instinct
(E) trial and error

Questions 3-5 :

(A) Imprinting
(B) Classical conditioning
(C) Fixed action pattern
(D) Altruism
(E) Operant conditioning

3. Gesst hatchlings follow the first thing they see


4. Innate, stereotypical behavior, which once begunm is continued to
completion no matter how useless or silly looking
5. Trial and error learning

148
Classified Real Questions
SAT II Biology E/M Dr Haitham AbdAllah
0100 36 777 19

6.Which of the following types of behavior describes the way that mice find their way through mazes?
(A) Habituation
(B) Imprinting
(C) Reasoning
(D) Instinct
(E) Trial and error

Answer Key :
1. A
2. A
3. A
4. C
5. E
6. E

149
Classified Real Questions
SAT II Biology E/M Dr Haitham AbdAllah
0100 36 777 19

Miscellaneous

1.
Which of the following is NOT a characteristic of bacteria?
(A) Circular double-stranded DNA
(B) Membrane-bound cellular organelles
(C) Plasma membrane consisting of lipids and proteins
(D) Ribosomes that synthesize polypeptides
(E) Cell wall made of peptidoglycan

2.
A feature of amino acids NOT found in carbohydrates is the presence of
(A) carbon atoms
(B) oxygen atoms
(C) nitrogen atoms
(D) hydrogen atoms
(E) phosphorus atoms

3. Which of the following is an anabolic reaction?


(A) ATP + H2O → ADP + Pi
(B) Cellular respiration
(C) Starch + n(H2O) → n(C6H12O6)
(D) CO2 + H2O → C6H12O6 + O2
(E) Glycolysis

Questions 4-6 :

150
Classified Real Questions
SAT II Biology E/M Dr Haitham AbdAllah
0100 36 777 19

The following questions refer to this sketch of an important structure in


any eukaryotic cell.

4. Where would this structure most likely be found?


(A) Golgi apparatus
(B) nucleus
(C) nucleolus
(D) chloroplast
(E) ribosome

5. The purpose of this structure is most directly related to which of the following?
(A) production of energy
(B) replication of DNA
(C) intake of materials into a cell
(D) cell division
(E) packaging substances for the cell

6. Which of the following would have the most profound effect on the normal functioning of this
mechanism?
(A) blocking the formation of microtubules
(B) blocking of the sarcoplasmic reticulum
(C) increasing the release of neurotransmitter
(D) blocking of the signal transduction pathway
(E) allowing protons to diffuse through the membrane

Questions 7-10 : refer to the following illustration and information

The cell cycle is a series of events in the life of a dividing eukaryotic cell. It consists of four stages:
G1, S, G2 , and M. The duration of the cell cycle varies from one species to another, and from one cell
type to another. The G1 phase varies the most. For example, embryonic cells can pass through the G1
phase so quickly that it hardly exists, whereas neurons are arrested in the cell cycle and do not divide

151
Classified Real Questions
SAT II Biology E/M Dr Haitham AbdAllah
0100 36 777 19

7. During which phase do chromosomes replicate?


(A) G1
(B) S
(C) G2
(D) M
(E) Cytokinesis

8. In mammalian cells, the first sign of prophase is the


(A) appearance of chromosomes
(B) separation of chromatids
(C) disappearance of the nuclear membrane
(D) replication of chromosomes
(E) crossing over of homologous chromosomes

9. Mitosis occurs in all of the following types of cells EXCEPT


(A) epidermal cells
(B) hair cells
(C) red blood cells
(D) pancreatic cells
(E) kidney cells

10. Since neurons are destined never to divide again, what conclusion can be made?
(A) These cells will go through cell division
(B) These cells will be permanently arrested in the G1 phase
(C) These cells will be permanently arrested in the G2 phase
(D) These cells will quickly enter the S-phase.
(E) The duration of the cell cycle will be long

Questions 11-13 :

Refer to the figure and chart below :

152
Classified Real Questions
SAT II Biology E/M Dr Haitham AbdAllah
0100 36 777 19

11.Which of the following DNA strands would serve as a template for the amino acid sequence shown
above?
(A) 3' -ATGCGACCAGCACGT- 5'

153
Classified Real Questions
SAT II Biology E/M Dr Haitham AbdAllah
0100 36 777 19

(B) 3' -AUGCCACUAGCACGU- 5'


(C) 3' -TACGGTGATCGTGCA- 5'
(D) 3' -UACGGUGAUCGUGCA-5'
(E) 3' -TGCACGATCACCGTA- 5'

12. lf a mutation occurs in which uracil is deleted from the messenger RNA after methionine is translated,
which of the following represents the resulting amino acid sequence?
(A) serine-histidine-serine-threonine
(B) methionine-proline-glutamine-histidine
(C) methionine-proline-leucine-alanine-arginine
(D) methionine-proline-alanine-arginine-arginine
(E) serine-proline-leucine-alanine-arginine

13. The mRNA above was found to be much smaller than the original mRNA synthesized in the nucleus.
This is due to the
(A) addition of a poly (A) tail to the mRNA molecule
(B) addition of a cap to the mRNA molecule
(C) excision of exons from the mRN A molecule
(D) excision of introns from the mRNA molecule
(E) translation of the mRNA molecule

Questions 14-16 :

In a laboratory experiment studying enzyme function, an enzyme is combined with a substrate and a
product forms as a precipitate. A spectrophotometer is used to measure the amount of precipitate that
forms. An increase in absorption indicates more product formed. Measurements are taken every minute.
Here are the data.

154
Classified Real Questions
SAT II Biology E/M Dr Haitham AbdAllah
0100 36 777 19

14. Regarding setup I, what is the most likely (D) I would use an enzyme that was
reason that the absorption does not change after not affected by temperature or
three minutes? pH
(A) The pH is not a favorable (E) I would control the variables
environment for this enzyme. better. The enzyme in setup IV
(B) The enzyme itself breaks down should be at the same pH as the
after four minutes pH of setup III.
(C) The data was not accurately
collected
(D) All of the substrate was broken
down after three minutes
(E) The enzyme was used up after
three minutes.

15. Which of the following statements is best


supported by the data?
(A) A pH of 7 is optimal for
this enzyme
(B) A pH of 4 is optimal for
this enzyme
(C) The enzyme has no
activity at pH 3
(D) This is most likely a
human enzyme
(E) This enzyme can
function at more than
one pH

16. If you were asked to carry out this


experiment again, what would you change?

(A) I would use more enzyme and


substrate to get better results
(B) I would take measurements
every minute up to ten minutes
to get a better idea of what is
going on
(C) I would use another instrument
to measure absorbance because
the spectrophotometer is not
accurate when a precipitate is
involved

155
Classified Real Questions
SAT II Biology E/M Dr Haitham AbdAllah
0100 36 777 19

Questions 17-19 :
An experiment was designed to determine the toxicity of the wood preservative
pentachlorophenol. Mosquito larvae were placed in beakers of water containing different
concentrations of pentachlorophenol.
The larvae were observed and the number alive was determined at the beginning of the experiment
and after 24 hours.
The percent of larvae remaining alive after 24 hours is shown in the graph below, which represents
the average of 3 replicates of each treatment

17.which statement is best supported by the data?


(A) Pentachlorophenol is toxic to mosquito larvae, even in very small amounts
(B) Pentachlorophenol is lethal to all aquatic lives
(C) Pentachlorophenol acts by blocking cell respiration reactions
(D) Pentachlorophenol preserves wood by killing fungi and bacteria that cause decay
(E) The toxicity of pentachlorophenol is inversely proportional to concentration

18. The experimental design includes all of the following EXCEPT


(A) Multiple trials
(B) A control population

156
Classified Real Questions
SAT II Biology E/M Dr Haitham AbdAllah
0100 36 777 19

(C) Dose effect


(D) Biological magnification
(E) Survival measurements

19. The role of the beaker containing 0 parts per million of pentachlorophenol is to
(A) Give an estimate of the number of larvae that die without pentachlorophenol
(B) Determine the number of larvae that become adults
(C) Determine whether water already contains pentachlorophenol
(D) Determine the growth rate of larvae in water
(E) Provide a supply of larvae to be used in later experiments

20.
It is important for humans to eat vegetables and fruits that are yellow-orange
because these foods
(A) Are rich in the ten essential amino acids
(B) Prevent color blindness
(C) Are the main source of vitamin D
(D) Provide carotene from which vitamin A is synthesized
(E) Contain pigments necessary for the production of hemoglobin

Answer Key ( Miscellaneous ) :


1. B
2. C
3. D
4. D
5. A
6. E
7. B
8. A .. One of the first signs of prophase in mammalian cells is the appearance of chromosomes.
9. C … Mitosis occurs in all of the following type of cells except mature red blood cells. Mature
red blood cells are short-lived and do not divide
10. B .. Because neurons are not capable of dividing_ it is reasonable to conclude that these
cells will not complete the G1 phase. This is a reading comprehension question. The passage
states that cells that do not divide are arrested at the G1 phase. (A), These cells will not be
committed to go through cell division. (C) and (D), These cells will not enter the G2 or S-
phase. (E), The duration of the cell cycle would be brief, not long
11. C
12. B
13. D
14. D …. after three minutes, there was no more substrate to break down. If the pH were not
favorable, there would not have been any reaction from the beginning. There is no reason that
the enzyme would break down or was used up. Enzymes do not get used up; they are reused

157
Classified Real Questions
SAT II Biology E/M Dr Haitham AbdAllah
0100 36 777 19

and unchanged in any reaction. The data makes sense and there is no evidence that it was not
collected correctly
15. E … From the data, you can see that the enzyme functions at pH 7, pH 4, and pH 2. There is
no data about what is optimum for this enzyme as choices A and B state. There is also no
evidence that this is a human enzyme
16. E …. There are too many variables in this experiment as performed. One variable is fresh or
boiled enzyme. Another variable is pH. The spectrophotometer measures the clarity of a
mixture. It is the perfect machine to use for this experiment
17. A
18. D
19. C
20. D

158
Classified Real Questions

Das könnte Ihnen auch gefallen